Sei sulla pagina 1di 44

1.MAGALLONA v. ERMITA G.R.

187167, August 16, 2011 Philippinearchipelago and classified adjacent terr itories such as the Kalayaan IslandGround (KIG) and
the Scarborough Shoal as “regimes of islands” whose islands generate their own applicable maritime
zones .Petitioners, in their capacities as “citizens, taxpayers or legislators” assail the constitutionality of
Facts: R.A. 9522 with one of their arguments contending that the law unconstitutionally “converts” internal
waters into archipelagic waters, thus subjecting these waters to the right of innocent and sea lanes
In 1961, Congress passed R.A. 3046 demarcating the maritime baselines of the Philippines as an passage under UNCLOS III, including overflight. Petitioners have contended that these passage rights
Archepelagic State pursuant to UNCLOS I of 9158, codifying the sovereignty of State parties over their will violate the Constitution as it shall expose Philippine internal waters to nuclear and maritime
territorial sea. Then in 1968, it was amended by R.A. 5446, correcting some errors in R.A. 3046 pollution hazard.
reserving the drawing of baselines around Sabah. In 2009, it was again amended by R.A. 9522, to be
compliant with the UNCLOS III of 1984. The requirements complied with are: to shorten one baseline, ISSUE:
to optimize the location of some basepoints and classify KIG and Scarborough Shoal as ‘regime of
Whether or not R.A. 9522 is unconstitutional for converting internal waters into archipelagic waters
islands’. Petitioner now assails the constitutionality of the law for three main reasons: 1. it reduces the
Philippine maritime territory under Article 1; 2. it opens the country’s waters to innocent and sea lanes HELD:
passages hence undermining our sovereignty and security; and 3. treating KIG and Scarborough as
Petition DISMISSED. The Court finds R.A. 9522 constitutional and is consistent w ith thePhilippine’s
‘regime of islands’ would weaken our claim over those territories. Issue: Whether R.A. 9522 is
national interest. Aside from being a vital step in safeguarding thecountry’s maritime zones, the law also
constitutional? Ruling: 1. UNCLOS III has nothing to do with acquisition or loss of territory. it is just a
allows an i nternationally-recognizeddelimitation of the breadth of the Philippine’s maritime zones and
codified norm that regulates conduct of States. On the other hand, RA 9522 is a baseline law to mark out
continentalshelf. The Court also finds that the conversion of internal waters intoarchipelagic waters will
basepoints along coasts, serving as geographic starting points to measure. it merely notices the
not risk the Philippines as affirmed in the Article 49 of the UNCLOS III, an archipelagic State has
international community of the scope of our maritime space. 2. If passages is the issue, domestically, the
sovereign power that extends to thewaters enclosed by the archipelagic baselin es, regardless of their
legislature can enact legislation designating routes within the archipelagic waters to regulate innocent
depth ordistance from the coast. It is further stated that the regime of archipelagic sea lanes passage will
and sea lanes passages. but in the absence of such, international law norms operate. the fact that for
not affect the status of its archipelagic waters or the exercise of sovereignty over waters and air space,
archipelagic states, their waters are subject to both passages does not place them in lesser footing vis a
bed and subsoil and the resources therein.
vis continental coastal states. Moreover, RIOP is a customary international law, no modern state can
invoke its sovereignty to forbid such passage. 3. On the KIG issue, RA 9522 merely followed the 2.Oposa vs Factoran
basepoints mapped by RA 3046 and in fact, it increased the Phils.’ total maritime space. Moreover, the
itself commits the Phils.’ continues claim of sovereignty and jurisdiction over KIG. If not, it would be a Natural and Environmental Laws; Constitutional Law: Intergenerational Responsibility GR No. 101083;
breach to 2 provisions of the UNCLOS III: Art. 47 (3): ‘drawing of basepoints shall not depart to any July 30 1993
appreciable extent from the general configuration of the archipelago’. Art 47 (2): the length of baselines
FACTS: A taxpayer’s class suit was filed by minors Juan Antonio Oposa, et al., representing their
shall not exceed 100 mm. KIG and SS are far from our baselines, if we draw to include them, we’ll
generation and generations yet unborn, and represented by their parents against Fulgencio Factoran Jr.,
breach the rules: that it should follow the natural configuration of the archipelago.
Secretary of DENR. They prayed that judgment be rendered ordering the defendant, his agents,
PROF. MERLIN M. MAGALLONA, et.al v HON. EDUARDO ERMITA, IN HISCAPACITY AS representatives and other persons acting in his behalf to:
EXECUTIVE SECRETARY, et.al G.R. No. 187167, 16 July 2011, EN BANC (Carpio, J.) The
1. Cancel all existing Timber Licensing Agreements (TLA) in the country;
conversion of internal waters into archipelagic waters will not risk the Philippines because an
archipelagic State has sovereign power that extends to the waters enclosed by the archipelagic baselines, 2. Cease and desist from receiving, accepting, processing, renewing, or appraising new
regardless of their depth or distance from the coast. TLAs;

R.A. 9522 was enacted by the Congress in March 2009 to comply with the terms of the United Nations and granting the plaintiffs “such other reliefs just and equitable under the premises.” They alleged that
Convention on the Law of the Sea (UNCLOS III),which the Philippines ratified on February 27, 1984. they have a clear and constitutional right to a balanced and healthful ecology and are entitled to
Such compliance shortened one baseline, optimized the location of some base points around the protection by the State in its capacity as parens patriae. Furthermore, they claim that the act of the

1
defendant in allowing TLA holders to cut and deforest the remaining forests constitutes a them from the Spanish Crown. He tried twice to have it registered during the Spanish occupation but to
misappropriation and/or impairment of the natural resources property he holds in trust for the benefit of no avail. In 1901 he filed a petition alleging ownership of the land but he was only granted a possessory
the plaintiff minors and succeeding generations. The defendant filed a motion to dismiss the complaint title.
on the following grounds:
- Premilinary issues. o Whether the mode of reaching the US supreme court was right (this was a writ of
1. Plaintiffs have no cause of action against him; 2. The issues raised by the error, some were saying that it should have been an appeal) – Holmes said that the mode was correct.
plaintiffs is a political question which properly pertains to the legislative or executive branches of the Writ of error was the general rule, appeal is the exception. He saw no reason not to apply the general rule
government. to this case.

ISSUE: Do the petitioner-minors have a cause of action in filing a class suit to “prevent the o Another issue was that even if Carino was able to have a title over the land, he could not have it
misappropriation or impairment of Philippine rainforests?” registered because Benguet was one of the excluded provinces in the Philippine Commission’s act no.
926 (AN ACT PRESCRIBING RULES AND REGULATIONS GOVERNING THE
HELD: Yes. Petitioner-minors assert that they represent their generation as well as generations to come.
HOMESTEADING, SELLING, AND LEASING OF PORTIONS OF THE PUBLIC DOMAIN OF THE
The Supreme Court ruled that they can, for themselves, for others of their generation, and for the
PHILIPPINE ISLANDS…). But that law dealt with acquisition of new titles and perfecting of titles
succeeding generation, file a class suit. Their personality to sue in behalf of succeeding generations is
begun under the Spanish law. Carino argued that he could register the land under Philippine
based on the concept of intergenerational responsibility insofar as the right to a balanced and healthful
Commissions Act no. 496 which covered the entire Philippine archipelago. Holmes held that he could
ecology is concerned. Such a right considers the “rhythm and harmony of nature” which indispensably
register the land if ownership can be maintained - Main issue: whether Carino owns the land.
include, inter alia, the judicious disposition, utilization, management, renewal and conservation of the
country’s forest, mineral, land, waters, fisheries, wildlife, offshore areas and other natural resources to
the end that their exploration, development, and utilization be equitably accessible to the present as well
o Government’s argument: Spain had title to all the land in the Philippines except those it saw fit to
as the future generations. Needless to say, every generation has a responsibility to the next to preserve
permit private titles to be acquired. That there was a decree issued by Spain that required registration
that rhythm and harmony for the full enjoyment of a balanced and healthful ecology. Put a little
within a limited time. Carino’s land wasn’t registered and so in effect it became public land. USSC:
differently, the minor’s assertion of their right to a sound environment constitutes at the same time, the
Whatever the position of Spain was on the issue, it does not follow that the US would view plaintiff to
performance of their obligation to ensure the protection of that right for the generations to come.
have lost all his rights to the land – this would amount to a denial of native titles throughout Benguet just
because Spain would not have granted to anyone in the province the registration of their lands. Organic
act of July 1, 1902 provides that all the property and rights acquired there by the US would be for the
3.CARINO V INSULAR GOVT
benefit of the inhabitants thereof. This same statute made a bill of rights embodying the safeguards of the
Mateo Carino (plaintiff in error) vs. Insular Government of the Philippines (defendant in error) 212 US constitution, it provides that “'no law shall be enacted in said islands which shall deprive any person of
449, 41 Phil Justice Holmes life, liberty, or property without due process of law, or deny to any person therein the equal protection of
the laws”. It would be hard to believe that that “any person” didn’t include the inhabitants of Benguet.
How it reached the court: - Plaintiff applied for registration of a certain land. Initially it was granted by Nor it meant “property” to refer only to those lands which had become such under a ceremony(of
the court, but the Government of the Philippines and the government of the United states appealed to the registration) many of the people of the land may have not even heard of. Although in sec. 14 of the
Court of first instance of Benguet (they were taking the property for public and military purposes. The organic act, it is said that the Philippine commission may prescribe rules and regulations for perfecting
CFI dismissed the application (for registration) and this was affirmed by the Philippine Supreme Court. titles to public lands, it should be noted that this section refers to those cases where the land was
This was brought to the US Supreme court by writ of error. admitted to be public land. The US SC hesitates to suppose that it was intended to declare every native
who had not a paper title, a trespasser.
Facts: - Plaintiff, an Igorot, possessed the land for more than 30 years before the treaty of Paris. He and
his ancestors had held the land for years. The local community recognizes them as the owners of the said The question still remains: what property and rights did the US acquire? in cases like this one, the
land. His grandfather lived upon it and maintained fences around the property. His father raised cattle on presumption would and should be against the government. As far back as memory goes, the land has
the property and he had inherited the land according to Igorot custom. Although no title was issued to been held by individuals under a claim of private ownership , it was never public land. It would not be

2
proper to just let the conqueror to dictate how to deal with the Philippine tribes if it really meant to use by TLA No. 87 was re -awarded to Twin Peaks Development and Realty Corporation under TLA No.
the rights acquired by them “for the benefit of the inhabitants thereof”. The natives were recognized by 356 which was set to expire on July 31, 2009, while the other half was allowed to be logged by Filipinas
the Spanish laws to own some lands, irrespective of any royal grant. They didn’t intend to turn all the Loggers, Inc. without the benefit of a formal award or license; and, That the latter entities were
inhabitants into trespassers. Principle of prescription was admitted: that if they weren’t able to produce controlled or owned by relatives or cronies of deposed President Ferdinand Marcos. The MNR issued an
title deeds, it is sufficient if they show ancient possession, as a valid title by prescription. Although there order denying petitioner's request. It ruled that a timber license was not a contract within the due process
was a decree in June 25, 1880 that required everyone to get a document of title or else lose his land, it clause of the Constitution, but only a privilege which could be withdrawn whenever public interest or
does not appear that it meant to apply to all but only those who wrongfully occupied royal lands. IT welfare so demands, and that
doesn’t appear that the land of Carino was considered as Royal land nor was it considered to have been
petitioner was not discriminated against in view of the fact that it was among ten concessionaires whose
wrongfully occupied. Two articles of the same decree provided that titles would be attributed to those
licenses were revoked in 1983. Moreover, emphasis was made of the total ban of logging operations in
who may prove possession for the necessary time. There were indications that registration was expected
the provinces of Nueva Ecija, Nueva Vizcaya, Quirino and Ifugao reasoning that the Ministry imposed
but it didn’t mean that ownership actually gained would be lost. The effect of the proof was not to confer
the ban because it realizes the great responsibility it bear [sic] in respect to forests. It considers itself the
title to them but to establish it. o “Law and justice require that the applicant should be granted what he
trustee thereof. This being the case, it has to ensure the availability of forest resources not only for the
seeks and should not be deprived of what, by the practice and belief of those among whom he lived, was
present, but also for the future generations of Filipinos. P’s MR denied. Meanwhile, MNR issued an
his property, through a refined interpretation of an almost forgotten law of Spain. “ Judgment reversed
administrative order lifting the logging ban in the province of Quirino. P then appealed to the Office of
the President which however denied it on the ground of lack of merit. It ruled that the appeal of
petitioner was prematurely filed, the matter not having been terminated in the MNR. Petitioner's motion
4.YSMAEL V EXEC SECRETARY DIGEST for reconsideration was denied. Hence, this petition for certiorari.
FACTS:

Soon after the change in government in 1986, Petitioner a letter to the Office of the President, and to
Issue: WON public respondent committed grave abuse of discretion amounting to lack or excess of
Minister Ernesto Maceda of the Ministry of Natural Resources [MNR], seeking: (1) the reinstatement of
jurisdiction and WON TLA are contracts.
its timber license agreement which was cancelled during the Marcos administration; (2) the revocation of
TLA No. 356 which was issued to Twin Peaks Development and Realty Corporation without public
bidding and in violation of forestry laws, rules and regulations; and, (3) the issuance of an order allowing
Held: NO. It is an established doctrine in this jurisdiction that the decisions and orders of administrative
petitioner to take possession of all logs found in the concession area. P alleged: it entered into a timber
agencies have upon their finality, the force and binding effect of a final judgment within the purview of
license agreement with the DENR, wherein it was issued an exclusive license to cut, collect and remove
the doctrine of res judicata. These decisions and orders are as conclusive upon the rights of the affected
timber except prohibited species within a specified portion of public forest land located in the
parties as though the same had been rendered by a court of general jurisdiction. The rule of res judicata
municipality of Maddela, province of Nueva Vizcaya from October 12, 1965 until June 30, 1990; That
thus forbids the reopening of a matter once determined by competent authority acting within their
on August 18, 1983, the Director of the Bureau of Forest Development [hereinafter referred to as
exclusive jurisdiction. As gleaned from the record, petitioner did not avail of its remedies under the law,
"Bureau"], issued a memorandum order stopping all logging operations in Nueva Vizcaya and Quirino
i.e. Section 8 of Pres. Dec. No. 705 as amended, for attacking the validity of these administrative actions
provinces, and cancelling the logging concession of petitioner and nine other forest concessionaires,
until after 1986. By the time petitioner sent its letter dated April 2, 1986 to the newly appointed Minister
pursuant to presidential instructions and a memorandum order of the Minister of Natural Resources That
of the MNR, requesting reconsideration of the above Bureau actions, these were already settled matters
on August 25, 1983, petitioner received a telegram from the Bureau, requesting him to stop all logging
as far as petitioner was concerned. Public respondents herein, upon whose shoulders rests the task of
operations and to pull out logging machineries and equipment in order to conserve remaining forests.
implementing the policy to develop and conserve the country's natural resources, have indicated an
That after the cancellation of its timber license agreement, it immediately sent a letter addressed to then
ongoing department evaluation of all timber license agreements entered into, and permits or licenses
President Ferdinand Marcos which sought reconsideration of the Bureau' s directive, citing in support
issued, under the previous dispensation. The ongoing administrative reassessment is apparently in
thereof its contributions to forest conservation and alleging that it was not given the opportunity to be
response to the renewed and growing global concern over the despoliation of forest lands and the utter
heard prior to the cancellation of its logging operation, but no favorable action was taken on this letter;
disregard of their crucial role in sustaining a balanced ecological system. The legitimacy of such concern
That barely one year thereafter, approximately one-half or 26, 000 hectares of the area formerly covered
3
can hardly be disputed, most especially in this country. The Court takes judicial notice of the profligate Office of the Mayor, through Ruzol, shall issue a permit to transport after payment of the corresponding
waste of the country's forest resources which has not only resulted in the irreversible loss of flora and fees to the municipal treasurer. • From 2001 to 2004, two hundred twenty-one (221) permits to transport
fauna peculiar to the region, but has produced even more disastrous and lasting economic and social salvaged forest products were issued to various recipients, of which forty-three (43) bore the signature of
effects. The delicate balance of nature having been upset, a vicious cycle of floods and droughts has been Ruzol while the remaining one hundred seventy-eight (178) were signed by his coaccused Guillermo T.
triggered and the supply of food and energy resources required by the people seriously depleted. While Sabiduria (Sabiduria), then municipal administrator of General Nakar. • 221 Informations for violation of
there is a desire to harness natural resources to amass profit and to meet the country's immediate Art. 177 of the RPC or for Usurpation of Authority or Official Functions were filed against Ruzol and
financial requirements, the more essential need to ensure future generations of Filipinos of their survival Sabiduria. o Claimed that the authority to issue such permits belonged to the DENR and not to the Office
in a viable environment demands effective and circumspect action from the government to check further of the Mayor. • Ruzol’s defense: o As Chief Executive of the municipality of General Nakar, Quezon, he
denudation of whatever remains of the forest lands. Nothing less is expected of the government, in view is authorized to issue permits to transport forest products pursuant to RA 7160 which give the LGU not
of the clear constitutional command to maintain a balanced and healthful ecology. Section 16 of Article only express powers but also those powers that are necessarily implied from the powers expressly
II of the 1987 Constitution. A long line of cases establish the basic rule that the courts will not interfere granted as well as those that are necessary, appropriate or incidental to the LGU’s efficient and effective
in matters which are addressed to the sound discretion of government agencies entrusted with the governance. Invoked the General Welfare Clause (§16, LGC) o RA 7160 has devolved certain functions
regulation of activities coming under the special technical knowledge and training of such agencies and responsibilities of the DENR to the LGU. And the permits to transport were issued pursuant to the
Petitioner is precluded from availing of the benefits of a writ of certiorari in the present case because he devolved function to manage and control communal forests with an area not exceeding fifty (50) square
failed to file his petition within a reasonable period. In the case at bar, petitioner waited for at least three kilometers. o Under (a) Section 5, Article X of the Constitution, (b) Section 129, Chapter I, Title One
years before it finally filed a petition for certiorari with the Book II of R.A. 7160, and (c) Section 186, Article Five, Chapter 5, Tile One, Book II of R.A. 7160, the
municipality is granted the power to create its own sources of revenue and to levy fees in accordance
Court attacking the validity of the assailed Bureau actions in 1983 and 1984. Considering that petitioner,
therewith. o The only kind of document the DENR issues relating to log, timber or lumber is
throughout the period of its inaction, was not deprived of the opportunity to seek relief from the courts
denominated “Certificate of Timber Origin” or CTO for logs and “Certificate of Lumber Origin” or CLO
which were normally operating at the time, its delay constitutes unreasonable and inexcusable neglect,
for lumber. o No proof of conspiracy between the two accused. o The DENR directly sanctioned and
tantamount to laches. Accordingly, the writ of certiorari requiring the reversal of these orders will not lie.
expressly authorized the issuance of the 221 Transport permits through the Provincial Environment and
A cursory reading of the assailed orders issued by public respondent Minister Maceda of the MNR,
natural Resources officer Rogelio Delgado Sr., in a Multi-Sectoral Consultative Assembly. •
which were affirmed by the Office of the President, will disclose public policy considerations which
Sandiganbayan: Acquitted Sabiduria but found Ruzol guilty as charged. o Cited §5, PD 705 (Forestry
effectively forestall judicial interference in the case at bar. Timber licenses, permits and license
Code): [The DENR] shall be responsible for the protection, development,
agreements are the principal instruments by which the State regulates the utilization and disposition of
forest resources to the end that public welfare is promoted. And it can hardly be gainsaid that they management, regeneration, and reforestation of forest lands; the regulation and supervision of the
merely evidence a privilege granted by the State to qualified entities, and do not vest in the latter a operation of licensees, lessees and permittees for the taking or use of forest products therefrom or the
permanent or irrevocable right to the particular concession area and the forest products therein. They occupancy or use thereof… o Likewise invoked EO 192 (Reorganizing the DENR). o Finally, citing RA
may be validly amended, modified, replaced or rescinded by the Chief Executive when national interests 7160, determined that since the authority relative to salvaged forest products was not included in the
so require. Thus , they are not deemed contracts within the purview of the due process of law clause above enumeration of devolved functions, the correlative authority to issue transport permits remains
Dismissed. with the DENR, and thus cannot be exercised by the LGUs.

5.RUZOL V. SANDIGANBAYAN ISSUES/HELD:

FACTS: • Leovegildo R. Ruzol was the mayor of General Nakar, Quezon from 2001 to 2004. • Earlier 1. WoN the authority to monitor and regulate the transportation of salvaged forest product is solely with
in his term, he organized a Multi-Sectoral Consultative Assembly composed of civil society groups, the DENR, and no one else. (NO)
public officials and concerned stakeholders with the end in view of regulating and monitoring the
2. WoN the permits to transport issued by Ruzol are valid. (NO)
transportation of salvaged forest products within the vicinity of General Nakar. • At the organizational
meeting for the assembly, the participants agreed that to regulate the salvaged forests products, the 3. WoN Ruzol is guilty of usurpation of official functions. (NO) RATIO:
4
in the management and control of communal forests within its territorial jurisdiction, We reiterate that
this authority should be exercised and enforced in accordance with the procedural parameters established
1. The LGU also has, under the LGC of 1991, ample authority to promulgate rules, regulations and
by law for its effective and efficient execution. • §17, LGC provides that the LGU’s authority to manage
ordinances to monitor and regulate salvaged forest products, provided that the parameters set forth by
and control communal forests should be “pursuant to national policies and is subject to supervision,
law for their enactment have been faithfully complied with. • While the DENR is, indeed, the primary
control and review of DENR.” • Before an area may be considered a communal forest, the following
government instrumentality charged with the mandate of promulgating rules and regulations for the
requirements must be accomplished: (1) an identification of potential communal forest areas within the
protection of the environment and conservation of natural resources, it is not the only government
geographic jurisdiction of the concerned city/municipality; (2) a forest land use plan which shall
instrumentality clothed with such authority. • While the law has designated DENR as the primary
indicate, among other things, the site and location of the communal forests; (3) a request to the DENR
agency tasked to protect the environment, it was not the intention of the law to arrogate unto the DENR
Secretary through a resolution passed by the Sangguniang Bayan concerned; and (4) an administrative
the exclusive prerogative of exercising this function. Whether in ordinary or in legal parlance, the word
order issued by DENR Secretary declaring the identified area as a communal forest. • In the present case,
“primary” can never be taken to be synonymous with “sole” or “exclusive.” • The General Welfare
the records are bereft of any showing that these requirements were complied with.
Clause of the LGC states: Every local government unit shall exercise the powers expressly granted, those
necessarily implied therefrom, as well as powers necessary, appropriate, or incidental for its efficient and
effective governance, and those which are essential to the promotion of the general welfare. Within their
3. Razol’s guilt was not proven beyond reasonable doubt. • Art. 177 of the RPC provides: Usurpation of
respective territorial jurisdictions, local government units shall ensure and support, among other things,
authority or official functions. — Any person who shall knowingly and falsely represent himself to be an
the preservation and enrichment of culture, promote health and safety, enhance the right of the people to
officer, agent or representative of any department or agency of the Philippine Government or of any
a balanced ecology, encourage and support the development of appropriate and self-reliant scientific and
foreign government, or who, under pretense of official position, shall perform any act pertaining to any
technological capabilities, improve public morals, enhance economic prosperity and social justice,
person in authority or public officer of the Philippine Government or any foreign government, or any
promote full employment among their residents, maintain peace and order, and preserve the comfort and
agency thereof, without being lawfully entitled to do so, shall suffer the penalty of prision correccional in
convenience of their inhabitants. • Pursuant to the aforequoted provision, municipal governments are
its minimum and medium periods. • There are two ways of committing the crime: o First, by knowingly
clothed with authority to enact such ordinances and issue such regulations as may be necessary to carry
and falsely representing himself to be an officer, agent or representative of any department or agency of
out and discharge the responsibilities conferred upon them by law, and such as shall be necessary and
the Philippine Government or of any foreign government. (Usurpation of authority) o Second, under
proper to provide for the health, safety, comfort and convenience, maintain peace and order, improve
pretense of official position, shall perform any act pertaining to any person in authority or public officer
public morals, promote the prosperity and general welfare of the municipality and its inhabitants, and
of the Philippine Government or any foreign government, or any agency thereof, without being lawfully
ensure the protection of property in the municipality. • There is a clear merit to the view that the
entitled to do so. (Usurpation of official functions) • In the present case, Ruzol stands accused of
monitoring and regulation of salvaged forest products through the issuance of appropriate permits is a
usurpation of official functions for issuing 221 permits to transport salvaged forest products under the
shared responsibility which may be done either by DENR or by the LGUs or by both. o DAO 1992-30:
alleged “pretense of official position and without being lawfully entitled to do so, such authority properly
LGUs shall share with the national government, particularly the DENR, the responsibility in the
belonging to the Department of Environment and Natural Resources.” • However, contrary to the ruling
sustainable management and development of the environment and natural resources within their
of the Sandiganbayan, We find that a careful scrutiny of the events surrounding this case failed to prove
territorial jurisdiction.
that Ruzol is guilty beyond reasonable doubt of committing the crime of usurpation of official functions
of the DENR. o We note that this case of usurpation against Ruzol rests principally on the prosecution’s
theory that the DENR is the only government instrumentality that can issue the permits to transport
2. The Permits to Transport issued by Ruzol are invalid for his failure to comply with the procedural
salvaged forest products. o But erstwhile discussed at length, the DENR is not the sole government
requirements set forth by law for its enforcement. • Ruzol insists that the permits partake of the nature of
agency vested with the authority to issue permits relevant to the transportation of salvaged forest
transport fees levied by the municipality for the use of public roads. • Ruzol is correct to
products, considering that, pursuant to the general welfare clause, LGUs may also exercise such
a point. o Nevertheless, We find that an enabling ordinance is necessary to confer the subject permits authority. • Moreover, Ruzol acted in good faith. o The conduct of a public consultation was a sign
with validity. o As correctly held by the Sandiganbayan, the power to levy fees or charges under the supporting Ruzol’s good intentions to regulate and monitor the movement of salvaged forest products to
LGC is exercised by the Sangguniang Bayan through the enactment of an appropriate ordinance wherein prevent abuse and occurrence of untoward illegal logging. DISPOSITION: Acquitted
the terms, conditions and rates of the fees are prescribed. • Although We recognize the LGU’s authority
5
6. CRISOSTOMO B. AQUINO vs. MUNICIPALITY OF MALAY AKLAN G.R. No. 211356; the general welfare clause, property rights of individuals may be subjected to restraints and burdens in
September 29, 2014 order to fulfill the objectives of the government. Otherwise stated, the government may enact legislation
that may interfere with personal liberty, property, lawful businesses and occupations to promote the
DOCTRINE: Based on law and jurisprudence, the office of the mayor has quasi-judicial powers to
general welfare. Under the law, insofar as illegal constructions are concerned, the mayor can, after
order the closing and demolition of establishments. This power granted by the LGC, as earlier explained,
satisfying the requirement of due notice and hearing, order their closure and demolition. One such piece
We believe, is not the same power devolved in favor of the LGU under Sec. 17 (b)(2)(ii ), as above-
of legislation is the LGC, which authorizes city and municipal governments, acting through their local
quoted, which is subject to review by the DENR. The fact that the building to be demolished is located
chief executives, to issue demolition orders. Under existing laws, the office of the mayor is given
within a forestland under the administration of the DENR is of no moment, for what is involved herein,
strictly speaking, is not an issue on environmental protection, conservation of natural resources, and the powers not only relative to its function as the executive official of the town; it has also been endowed
maintenance of ecological balance, but the legality or illegality of the structure. Rather than treating this with authority to hear issues involving property rights of individuals and to come out with an effective
as an environmental issue then, focus should not be diverted from the root cause of this debacle order or resolution thereon. Pertinent herein is Sec. 444 (b) (3) (vi) of the LGC, which empowered the
compliance. Facts: Petitioner is the president and chief executive officer of Boracay Island West Cove mayor to order the closure and removal of illegally constructed establishments for failing to secure the
Management Philippines, Inc. (Boracay West Cove). On January 7, 2010, the company applied for a necessary permits.
zoning compliance with the municipal government of Malay, Aklan.2 While the company was already
operating a resort in the area, and the application sought the issuance of a building permit covering the
construction of a three-storey hotel over a parcel of land measuring 998 sqm. located in Sitio Diniwid, 7. G.R. No. L-36847 July 20, 1983SERAFIN B. YNGSON vs. THE HON. SECRETARY OF
Barangay Balagab, Boracay Island, Malay, Aklan, which is covered by a Forest Land Use Agreement for AGRICULTURE and NATURAL RESOURCES, ANITA V. DEGONZALES and JOSE M.
Tourism Purposes (FLAgT) issued by the Department of Environment and Natural Resources (DENR) in LOPEZ
favor of Boracay West Cove. Through a Decision on Zoning dated January 20, 2010, the Municipal
Zoning Administrator denied petitioner’s application on the ground that the proposed construction site Facts:
was within the “no build zone” demarcated in Municipal Ordinance 2000-131 (Ordinance). Petitioner
A number of people, including the petitioner and private respondents, applied for a permit to utilize 66
appealed the denial action to the Office of the Mayor but despite follow up, no action was ever taken by
hectares of mangrove swamps for fishpond purposes, but were not granted, the area still being
the respondent mayor. A Cease and Desist Order was issued by the municipal government, enjoining the
considered to be communal forest. When the area was released for said purpose, the three private parties
expansion of the resort, and on June 7, 2011, the Office of the Mayor of Malay, Aklan issued the assailed
in this case had conflicting claims on the same. Initially, the Bureau of Fisheries awarded the whole area
EO 10, ordering the closure and demolition of Boracay West Cove’s hotel. Petitioner filed a Petition for
in favor of petitioner. However, the Sec. of Agriculture and Natural Resources later ordered the divisi on
Certiorari with prayer for injunctive relief with the CA Alleging that the order was issued and executed
of the area into three portions,one part for each of the petitioner and the private respondents. Not
with grave abuse of discretion.
satisfied with the portion received, petitioner appealed the order.

Issue: Whether or not petitioner is entitled to the whole of the area concerned.
PETITIONER’s CONTENTION: The hotel cannot summarily be abated because it is not a nuisance
Held:
per se, given the hundred million peso-worth of capital infused in the venture. And the Municipality of
Malay, Aklan should have first secured a court order before proceeding with the demolition. No. It is elementary in the law governing the disposition of lands of the public domain that until timber
or forest lands are released as disposable and alienable neither the Bureau of Lands nor the Bureau of
RESPONDENT’s CONTENTION: The demolition needed no court order because the municipal
Fisheries has authority to lease, grant, sell, or otherwise dispose of these lands for homesteads, sales
mayor has the express power under the Local Government Code (LGC) to order the removal of illegally
patents, leases for grazing or other purposes, fishpond leases, and other modes of utilization. The Bureau
constructed buildings. Issue: Whether or not a judicial proceedings be conducted first before the LGU
of Fisheries has no jurisdiction to administer and dispose of swamplands or mangrove lands forming part
can order the closure and demolition of the property in question. Held: Generally, LGUs have no power
of the public domain while such lands are still classified as forest land or timberland and not released for
to declare a particular thing as a nuisance unless such a thing is a nuisance per se. Despite the hotel’s
fishery or other purposes. All the applications in this case were premature; therefore not one of the
classification as a nuisance per accidens, however, we still find in this case that the LGU may
nevertheless properly order the hotel’s demolition. This is because, in the exercise of police power and
6
applicants can claim to have a preferential right over another. The interpretation by the Office of the by the occupant. In effect, it precludes the government from enforcing the said provision as it
President was held to be an exercise of sound discretion which should not be disturbed. decides to reclassify lands as alienable and disposable. The land in question was found to be cocal in
nature, it having been planted with coconut trees now over fifty years old. The inherent nature of the
land but confirms its certification in 1980 as alienable, hence agricultural. There is no
8.REPUBLIC VS. CA AND NAGUIT impediment to the application of Section 14 (1) of the Property Registration Decree. Naguit had
the right to apply for registration owing to the continuous possession by her and her predecessors-in-
FACTS: interest of the land since 1945.
Corazon Naguit filed a petition for registration of title which seeks judicial confirmation of her
imperfect title over a parcel of land in Nabas, Aklan. It was alleged that Naguit and her predecessors-in-
interest have occupied the land openly and in the concept of owner without any objection from
any private person or even the government until she filed her application for registration. The
REPUBLIC VS. CA AND NAGUIT
MCTC rendered a decision confirming the title in the name of Naguit upon failure of Rustico Angeles to
appear during trial after filing his formal opposition to the petition. The Solicitor General, FACTS:
representing the Republic of the Philippines, filed a motion for reconsideration on the grounds that the
Respondent applies for registration of title to 4 parcels of land contending she is the owner of the said
property which is in open, continuous and exclusive possession must first be alienable. Naguit
land which she acquired from the LID Corporation which in turn acquired thes ame from persons who
could not have maintained a bona fide claim of ownership since the subject land was declared as
have been in possession thereof for more than 30 years. The Republic filed in opposition that said lands
alienable and disposable only on October 15, 1980. The alienable and disposable character of the land
belong to the public domain and not subject to private appropriation .ISSUE: Whether or not the land in
should have already been established since June 12, 1945 or earlier.
dispute as a forest land belonging to public domain may be appropriated as private property.
ISSUE:
Ruling: For a public forest land/reserves to be subject for private appropriation, it requires an express
Whether or not it is necessary under Section 14 (1) of the Property Registration Decree that the subject and positive act of the government that it will become a part of alienable and disposable agricultural
land be first classified as alienable and disposable before the applicant’s possession under a bona fide lands of public domain. Occupation in the concept of an owner cannot ripen into private ownership and
claim of ownership could even start. be registered to as a title.

RULING:

Section 14 (1) merely requires that the property sought to be registered as already alienable and 9. LAGUA VS CUSI, 160 SCRA 69 (1983)
disposable at the time the application for registration of title is filed. There are three requirements for
FACTS:
registration of title, (1) that the subject property is alienable and disposable; (2) that the applicants and
their predecessor-in-interest have been in open, continuous, and exclusive possession and occupation, A memorandum was issued preventing the passage of Plaintiff Laguas' hauling trucks loaded with logs
and; (3) that the possession is under a bona fide claim of ownership since June 12, 1945. There must for the Japanese vessel on the national highway loading towards where the vessel was berthed. In
be a positive act of the government through a statute or proclamation stating the intention of the State to compliance with this directive, the security force of Defendant Eastcoast closed the road to the use by
abdicate its exclusive prerogative over the property, thus, declaring the land as alienable and disposable. plaintiffs trucks and other equipments and effectively prevented their passage thereof while the vehicles
However, if there has been none, it is presumed that the government is still reserving the right to utilize and trucks of other people were curiously not disturbed and were allowed passess
the property and the possession of the land no matter how long would not ripen into ownership through
acquisitive prescription. To follow the Solicitor General’s argument in the construction of Section 14 on the same road. It resulted that the loading of logs on the M/S"Kyofuku Maru" was discontinued.
(1) would render the paragraph 1 of the said provision inoperative for it would mean that all lands When Plaintiffs Laguas were already resuming the hauling operations of their logs towards the Japanese
of public domain which were not declared as alienable and disposable before June 12, 1945 Vessel, again that same road, only the day before ordered by the BFD to be opened for use and passage
would not be susceptible to original registration, no matter the length of unchallenged possession by plaintiffs, was closed to them by Defendant Eastcoast's security men upon a radio message order of
Defendant Maglana. Eventhe vessel M/S "Kyofuku Maruwas" ordered by Defendant Maglana to untie
7
her anchor contraryto existing laws, rules and regulations of the Bureau of Customs and the Philippine opposition insofar as a portion of LotNo. 885containing 117,956 square meters was concerned and
Coastguard. Given no recourse in the face of the blatant and illegal closure of the road indefiance of prayed that title to said portion be confirmed and registered in his name.
BFD orders to the contrary by the Defendant East coast through the order of Defendant Maglana,
Issue:
Plaintiff Laguas had to depart post paste to Mati, Davao Oriental, from Baganga where the shipment and
the road closure were made, to seek the assistance of the PC thereat. The private respondents filed a Whether or not the lot in question can be subject of registration and confirmation of title in the name of
motion to dismiss argued that petitioner Daylinda Laguas has no capacity to sue as her name was not the private person
registered as an "agent" or "dealer" of logsin the Bureau of Forestry. The Court agrees with the
defendants that under the law, the Bureauo f Forest Development has the exclusive power to regulate the HELD:
use of logging road and to determine whether their use is in violation of laws. The opposition of the Director of Forestry was strengthened by the appellate court's finding that timber
licenses had to be issued to certain licensees and even Jose Amunategui himself took the trouble to ask
for a license to cut timber within the area. It was only sometimein1950 that the property was converted
ISSUE: into fishpond but only after a previous warning from the District Forester that the same could not be
done because it was classified as "publicforest ‖. A forested area classified as forest land of the public
Whether or not the Bureau of Forest Development has the jurisdiction to determine firstthe legality of domain does not lose such classification simply because loggers or settlers may have stripped it of its
closure of logging roads before the case be directed to the regular courts forest cover. "Forestlands" do not have to be on mountains or in out of the way places. Swampy areas
covered by mangrove trees, nipa palms, and other trees growing in brackish or sea water may also be
HELD:
classified as forest land. The possession of forest lands, no matter how long, cannot ripen into private
NO. P.D. No. 705 upon which the respondent court based its order does not vest any power in the ownership. Therefore, the lot in question never ceased to be classified as forestland of public domain.
Bureau of Forest Development to determine whether or not the closure of a logging road is legal or 11. MERIDA V PEOPLE (Natural Resources)
illegal and to make such determination a pre-requisite before an action for damages may be maintained. MERIDA V PEOPLE (DEFINITION OF TIMBER, AUTHORITY OF FOREST OFFICERS)
Moreover, the complaint instituted by the petitioners is clearly for damages based on the alleged illegal
closure of the logging road. Whether or not such closure was illegal is a matter to be established on the G.R. No. 158182
part of the petitioners and a matter to be disproved by the private respondents. This should appropriately June 12, 2008
be threshed out in a judicial proceeding. It is beyond the power and authority of the Bureau of Forest
Development to determine the unlawful closure of a passage way, much less award or deny the payment FACTS:
of damages based on such closure. Not every activity inside a forest area is subject to the jurisdiction of
on 23 December 1998, Tansiongco learned that petitioner cut a narra tree in the Mayod Property.
the Bureau of Forest Development. As we have held in Ateneo de ManilaUniversity v. Court of appeals
Tansiongco reported the matter to Florencio Royo (Royo), the punong barangay of Ipil. On 24
(145 SCRA 100, 110):
December 1998, 7 Royo summoned petitioner to a meeting with Tansiongco. When confronted during
the meeting about the felled narra tree, petitioner admitted cutting the tree but claimed that he did so with
the permission of one Vicar Calix (Calix) who, according to petitioner, bought the Mayod Property from
10. HEIRS OF AMUNATEGUI VS DIRECTOR OF FORESTRY, 126 SCRA 69 (1983) Tansiongco in October 1987 under a pacto de retro sale. Petitioner showed to Royo Calix's written
FACTS: authorization signed by Calix's wife.

There were two petitions for review on certiorari questioning the decision of the Court of Appeals which On 11 January 1999, Tansiongco reported the tree-cutting to the Department of Environment and Natural
declared the disputed property as forestland, not subject to titling in favor of private persons, Borre and Resources (DENR) forester Thelmo S. Hernandez (Hernandez) in Sibuyan, Romblon.
Amunategui. The Director of Forestry, through the Provincial Fiscalof Capiz, also filed an opposition to DECISION OF LOWER COURTS:
the application for registration of title claiming that the landwas mangrove swamp which was still
classified as forest land and part of the public domain. Another oppositor, Emeterio Bereber filed his * DENR forester: ordered petitioner not to convert the felled tree trunk into lumber.

8
On 26 January 1999, Tansiongco informed Hernandez that petitioner had converted the narra trunk into At any rate, Tansiongco was not precluded, either under Section 80 of PD 705 or the Revised Rules,
lumber. Hernandez, with other DENR employees and enforcement officers, went to the Mayod Property from filing a complaint before the Provincial Prosecutor for petitioner's alleged violation of Section 68 of
and saw that the narra tree had been cut into six smaller pieces of lumber. Hernandez took custody of the PD 705, as amended. For its part, the trial court correctly took cognizance of Criminal Case No. 2207 as
lumber, 9 deposited them for safekeeping with Royo, and issued an apprehension receipt to petitioner. A the case falls within its exclusive original jurisdiction.
larger portion of the felled tree remained at the Mayod Property. The DENR subsequently conducted an
2) Whether petitioner is liable for violation of Section 68 of PD 705, as amended.
investigation on the matter.

* RTC (upon complaint of Tansiongco): Petitioner was charged in the Regional Trial Court of Romblon,
Romblon, Branch 81 (trial court) with violation of Section 68 of PD 705, as amended, for "cut[ting], YES.
gather[ing], collect[ing] and remov[ing]" a lone narra tree inside a private land in Mayod, Ipil,
Magdiwang, Romblon (Mayod Property) over which private complainant Oscar M. Tansiongco
(Tansiongco) claims ownership.
Before his trial, petitioner consistently represented to the authorities that he cut a narra tree in the Mayod
* CA: affirmed trial court. Property and that he did so only with Calix's permission. However, when he testified, petitioner denied
cutting the tree in question. We sustain the lower courts' rulings that petitioner's extrajudicial admissions
ISSUES & RULINGS: bind him.
1) Whether the trial court acquired jurisdiction over Criminal Case No. 2207 even though it was based 3) Is the narra tree timber?
on a complaint filed by Tansiongco and not by a DENR forest officer; and
YES.
YES, DENR has jurisdiction.
The closest this Court came to defining the term "timber" in Section 68 was to provide that "timber,"
[NOTE: This dispositive no longer applicable since the Rules of Procedure for Environmental cases includes "lumber" or "processed log."
requires complaint to be filed first with the DENR, but the preliminary investigation is done by the
prosecutor] In other jurisdictions, timber is determined by compliance with specified dimensions or certain "stand
age" or "rotation age." In Mustang Lumber, Inc. v. Court of Appeals, this Court was faced with a similar
Section 80 of PD 705 provides in relevant parts: task of having to define a term in Section 68 of PD 705 - "lumber" - to determine whether possession of
lumber is punishable under that provision. In ruling in the affirmative, we held that "lumber" should be
SECTION 80. Arrest; Institution of criminal actions. - x x x x
taken in its ordinary or common usage meaning to refer to "processed log or timber,"
Reports and complaints regarding the commission of any of the offenses defined in this Chapter, not
We see no reason why, as in Mustang, the term "timber" under Section 68 cannot be taken in its common
committed in the presence of any forest officer or employee, or any of the deputized officers or officials,
acceptation as referring to "wood used for or suitable for building or for carpentry or joinery." Indeed,
shall immediately be investigated by the forest officer assigned in the area where the offense was
tree saplings or tiny tree stems that are too small for use as posts, panelling, beams, tables, or chairs
allegedly committed, who shall thereupon receive the evidence supporting the report or complaint.
cannot be considered timber.
If there is prima facie evidence to support the complaint or report, the investigating forest officer shall
Undoubtedly, the narra tree petitioner felled and converted to lumber was "timber" fit "for building or for
file the necessary complaint with the appropriate official authorized by law to conduct a preliminary
carpentry or joinery" and thus falls under the ambit of Section 68 of PD 705, as amended.
investigation of criminal cases and file an information in Court. (Emphasis supplied)

Here, it was not "forest officers or employees of the Bureau of Forest Development or any of the
deputized officers or officials" who reported to Hernandez the tree-cutting in the Mayod Property but
Tansiongco, a private citizen who claims ownership over the Mayod Property. Thus, Hernandez cannot
be faulted for not conducting an investigation to determine "if there is prima facie evidence to support
the complaint or report."
9
Topic: Doctrine of Primary Jurisdiction
ISSUE:
12. PROVIDENT TREE FARMS, INC. (PTFI) VS HON. BATARIO & AJ INTERNATIONL
CORP. (AJIC) Whether or not the Bureau of Customs holds jurisdiction in the matter of wood product
importation.
GR 92285 March 28, 1994
HELD:
DOCTRINE:
The only subject of this incentive is a ban against importation of wood and wood products which
Cases before the BOC must be fully fleshed out before it prior to elevating the issues to a regular court in
is to be enforced by Bureau of Customs since it has under the Tariff and Customs Code the exclusive
keeping with the exhaustion of administrative remedies. (primary jurisdiction)
original jurisdiction over seizure and forfeiture cases. To allow the regular court to direct the
FACTS: Commissioner is clearly an interference with the exclusive jurisdiction of the BOC.

 PTFI is a Phil corporation engaged in industrial tree planting. It supplies to a local match PTFI’c correspondence with the BOC contesting the legality of match importations may already
manufacturer solely for production of matches. take the nature of administrative proceedings the pendency of which would preclude the court from
 There’s a state policy to encourage qualified persons to engage in industrial tree plantation under interfering with it under the doctrine of primary jurisdiction.
Revised Forestry Code which provides a set of incentives to corporations like PTFI and is a
qualified ban against importation of wood. In Presidential Commission on Good Government v. Peña, 20 the court held that —
 Respondent, AJIC, imported matches from Indonesia which the BOC released which violates the
Revised Forestry Code’s ban of importing wood and wood-derivated products. . . . . under the "sense-making and expeditious doctrine of primary
jurisdiction . . . the courts cannot or will not determine a controversy involving a
RTC: question which is within the jurisdiction of an administrative tribunal, where the
question demands the exercise of sound administrative discretion requiring the special
 PTFI filed with the RTC of Manila a complaint for injunction and damages with prayer for a TRO knowledge, experience, and services of the administrative tribunal to determine
against Commissioner of Customs to prohibit the latter from importing matches. technical and intricate matters of fact, and a uniformity of ruling is essential to comply
 AJIC moved to dismiss the complaint alleging that: with the purposes of the regulatory statute administered (Pambujan Sur United Mine
o The Commissioner of Customs and not the regular court has exclusive jurisdiction to Workers v. Samar Mining Co., Inc., 94 Phil. 932, 941 [1954].)
determine the legality of an importation.
o The release of importations had rendered injunction moot and academic. In this era of clogged court dockets, the need for specialized administrative boards or
o The prayer for damages has no basis as the Commissioner’s acts are in accordance with commissions with the special knowledge, experience and capability to hear and
law. determine promptly disputes on technical matters or essentially factual matters, subject
o The complaint for injunction cannot stand it being only a provisional relief and not a to judicial review in case of grave abuse of discretion, has become well indispensable .
principal remedy. ...
 PTFI opposed the motion to dismiss.
 AJIC’s motion to dismiss was denied. The court cannot compel an agency to do a particular act or to enjoin such act which is within its
 AJIC filed a motion for reconsideration and the Court reconsidered and dismissed the case on the prerogative, except when in the exercise of its authority it gravely abuses or exceeds its jurisdiction. In the
ground that it had no jurisdiction to determine what are legal or illegal importations. case at bench, we have no occasion to rule on the issue of grave abuse of discretion or excess of jurisdiction
as it is not before us.
 PTFI seeks to set aside the order of respondent court and prays for the continuation of the hearing
of the case contending that what was brought before the trial court was a civil case for injunction Thus, the order of the RTC was affirmed and the petition for review was denied.
for the purpose of securing compliance with the provision of the RFC.
10
13. MOMONGAN V OMIPON removed, or possessed or abandoned, and all conveyances used either by land, water or
air in the commission of the offense and to dispose of the same in accordance with
At around 10:00 o'clock of November 14, 1992, police officers of the Municipality of Hinunangan, pertinent laws, regulations or policies on the matter.
Southern Leyte apprehended Dionisio Golpe while he was driving his truck loaded with illegally cut
lumber. The truck and logs were impounded. A complaint was filed against Basilio Cabig, the alleged There may be some facts that are not extant in the records which can only come out
owner of the logs. After conducting the preliminary investigation, respondent Judge Rafael B. Omipon during a formal investigation to better establish clear culpability or exoneration over
found that a prima facie case exists against Mr. Cabig but he ordered the release of the truck inasmuch as the respondent.
the owner/driver, Mr. Golpe, was not charged in the complaint.
In view thereof, and to give respondent an opportunity to clear himself, it is
Regional Director Augustus L. Momongan of the Department of Environment and Natural Resources respectfully recommended that this matter be referred to Acting Executive Judge
filed the instant complaint against respondent Judge alleging that his order releasing the truck used in the Leandro T. Loyao, Jr., RTC, Branch 26, San Juan, Southern Leyte, for investigation,
transport of illegally cut forest products violated Presidential Decree 705, as amended by Executive report and recommendation within sixty days from receipt of the records. 3
Order No. 277, Section 68 and 68-A1and Administrative Order No. 59, Series of 1990. 2 Complainant
claims that respondent Judge has no authority to order the release of the truck despite the non-inclusion In the Resolution of November 8, 1993, the Court resolved to refer the case to Acting Executive Judge
of Mr. Golpe in the complaint. The truck should have been turned over to the Community Environment Leandro T. Loyao, Jr., RTC, Branch 26, San Juan, Southern Leyte, for investigation, report and
and Natural Resources Office of San Juan, Southern Leyte for appropriate disposition as the same falls recommendation, within sixty (60) days from receipt of the records. 4
under the administrative jurisdiction of the Department of Environment and Natural Resources Office.
During the first two hearing dates, complainant was unable to attend but sent his representatives, DENR
In his comment, respondent Judge explained that after conducting the preliminary investigation, he lawyer Constantino Esber and legal assistant Romeo Gulong. Respondent Judge appeared with his
found that Golpe, the owner of the truck, is principally engaged in the hauling of sand and gravel and the counsel. However, on the third hearing date, respondent Judge failed to appear as he suffered a stroke
delivery of hollow blocks. On his way home after delivering hollow blocks in Barangay Sto. Niño II, he and was hospitalized. Thereafter, DENR counsel Esber manifested that their office has filed a motion for
met his friend Cabig who requested him to load sliced lumber and deliver the same at Brgy. Lungsod- reinvestigation and for the turnover of the jeep to the PNP and subsequently, to the DENR. He also
daan, Hinundayan to be used for the construction of a barangay high school building. They were manifested that the complainant is submitting the administrative matter for resolution and
apprehended when the truck had a flat tire. After changing the tire, both the lumber and the truck were recommendation without adducing evidence against respondent. Respondent's counsel did not object to
ordered deposited at the police station of Hinunangan. complainant's manifestation. The counsel of both complainant and respondent jointly agreed to submit
the case for appropriate action.
Respondent Judge observed that Golpe has a lesser participation in the crime of illegal logging and,
being a mere accessory, he might be utilized by the Acting Chief of Police as prosecution witness against The Investigating Judge's confidential report, in part, states:
Cabig. More importantly, the fact that the complaint charged only Cabig, respondent Judge, in the
exercise of his sound discretion, ordered the release of the truck owned by Golpe. In view of this development in the course of an intended investigation this investigator could not
elicit additional facts than are found in the records, whether inculpatory or exculpatory. Respondent
The Memorandum of the Office of the Court Administrator recommended that a formal investigation be was given an opportunity to explain the unfavorable circumstances against him but he was overtaken
conducted. An excerpt from its Memorandum states: by a serious illness. So much was expected from the complainant to supply the facts not extant in the
records, but he lost interest in substantiating his April 1993 report to the Supreme Court. In fact, he
We find the explanation of respondent unsatisfactory. While he is authorized to was submitting this administrative matter for resolution without adducing evidence against
conduct preliminary investigation in all cases of violations of P.D. 705, as amended, respondent.
otherwise known as the Revised Forestry Code of the Philippines, Sec. 68-A thereof
provides that it is the Department Head or his duly authorized representative who may Except for the 21 January 1994 motion for reinvestigation of DENR counsel Esber which sought for
order the confiscation and disposition of the forest products illegally cut, gathered, the inclusion of jeep owner and driver Dionisio Golpe in the criminal information, there is nothing
11
new that can be added to the facts found by the Honorable Deputy Court Administrator as reflected in 12 thereof categorically states that "[t]he confiscation of the conveyance under these regulations shall be
his Memorandum for the Honorable Chief Justice dated 12 October 1993. without prejudice to any criminal action which shall be filed against the owner thereof or any person
who used the conveyance in the commission of the offense."
There being no actual investigation conducted, no additional facts could be reported and
consequently, there is no basis for a recommendation on the basis of facts. Petitioner is of the opinion that under the circumstances, respondent Judge should have turned over the
truck to the Community Environment and Natural Resources Office (CENRO) of San Juan, Southern
This investigator can only recommend appropriate action by the Supreme Court on the basis of the Leyte for appropriate disposition. No doubt, this would have simplified matters and prevented the
facts already extant in the records with a prayer for consideration of respondent plight especially so present situation from occurring wherein one government official files a complaint against another.
since on account of this investigation his health has deteriorated and may affect his efficiency output Under Sec. 4 of Adm. Order No. 59, if the apprehension is not made by DENR field offices, deputized
as a judge. Perhaps, allowing him to bow out of the service with honor and corresponding benefits.5 military personnel and officials of other agencies apprehending illegal logs and other forest products and
their conveyances shall notify the nearest DENR field offices and turn over said forest products and
During the pendency of this case, respondent Judge filed for disability retirement. His application was conveyances for proper action and disposition. A period of about two weeks lapsed from the time the
approved but his pension was not released pending the outcome of this case. seizure was made before a complaint was filed. During this period, the apprehending policemen had
enough time to turn over the logs and the truck to the nearest DENR field office for proper action and
We find respondent Judge's order to release the truck owned and driven by Mr. Dionisio Golpe legally disposition since the duty to turn over the truck to the nearest DENR field office rests on the officials
justifiable, hence, he is not subject to any disciplinary sanction. apprehending the illegal logs. There being no mandatory duty on the part of respondent Judge to turn
over the truck, he should not be visited with disciplinary sanction when he did not refer the same to the
According to the Revised Penal Code, Art. 45, first paragraph: "[E]very penalty imposed for the DENR field office in San Juan, Southern Leyte.
commission of a felony shall carry with it the forfeiture of the proceeds of the crime and the instrument
or tools with which it was committed." However, this cannot be done if such proceeds and instruments or The Court takes this opportunity to enjoin the National Police, the DENR, the prosecutors, and the
tools "be the property of a third person not liable for offense." In this case, the truck, though used to members of the bench to coordinate with each other for a successful campaign against illegal logging. It
transport the illegally cut lumber, cannot be confiscated and forfeited in the event accused therein be behooves all the concerned agencies to seriously strive for the attainment of the constitutionally-declared
convicted because the truck owner/driver, Mr. Dionisio Golpe was not indicted. Hence, there was no policy to "protect and advance the right of the people to a balanced and healthful ecology in accord with
justification for respondent Judge not to release the truck. the rhythm and harmony of nature" 7 in order to preserve our natural resources for the benefit of the
generations still to come.
Complainant is correct in pointing out that based on Pres. Decree No. 705, Sec. 68-A and Adm. Order
No. 59, the DENR Secretary or his duly authorized representative has the power to confiscate any WHEREFORE, the complaint is DISMISSED.
illegally obtained or gathered forest products and all conveyances used in the commission of the offense
and to dispose of the same in accordance with pertinent laws. However, as complainant himself likewise 14. INTERNATIONAL HARDWOOD AND VENEER COMPANY OF THE PHILIPPINES,
pointed out, this power is in relation to the administrative jurisdiction of the DENR. petitionerappellee, vs. UNIVERSITY OF THE PHILIPPINES and JOSE C. CAMPOS, JR.,
respondentsappellants
We do not find that when respondent Judge released the truck after he conducted the preliminary
G.R. No. 52518. August 13, 1991
investigation and satisfied himself that there was no reason to continue keeping the truck, he violated
Pres. Decree No. 705 and Adm. Order No. 59. The release of the truck did not render nugatory the J. Davide, Jr.
administrative authority of the DENR Secretary. The confiscation proceedings under Adm. Order No.
596 is different from the confiscation under the Revised Penal Code, which is an additional penalty
imposed in the event of conviction. Despite the order of release, the truck can be seized again either by FACTS:
filing a motion for reinvestigation and motion to include the truck owner/driver, as co-accused, which
complainant has done as manifested before the lower court or by enforcing Adm. Order No. 59. Section Kind of action: a special civil action for declaratory relief with injunction

12
Effects of cession in full ownership of a land removed from public domain RTC Ruling: Ruling in favor of Hardwood. The demands of UP have no legal basis. The cession in full
ownership of the tract of land under RA 3990 was expressly made ‘subject to any existing concessions.’
Hardwood is engaged in the manufacture, processing and exportation of plywood and was, for said
Inasmuch as at the time of RA 3990, the Hardwood’s timber concession over the tract of land was existing
purpose, granted by the Government an exclusive license for a period of 25 years expiring on February 1,
and would continue to exist until February 1, 1985, UP will acquire ‘full ownership’ and exclusive
1985.
jurisdiction to control and administer the property only after February 1, 1985.
Sometime on September 25, 1961, during the effectivity of License Agreement President Garcia the issued
EO No. 791 thereby withdrawing from sale or settlement and reserving for the College of
Agriculture, University of the Philippines, as experiment station for the proposed Dairy Research and Arguments of opposing sides before the SC:
production studies of this College, a certain parcel of land of the Public domain situated partly in the
municipalities of Paete and Pakil, province of Laguna, and partly in the municipality of Infanta, Province UP HARDWOOD
of Quezon, subject to private rights, if any there be, and to the condition that the disposition of timber and A. Under RA 3990, UP may collect forest UP has not been granted by R.A. No. 3990 the
other forest products found therein shall be subject to the forestry laws and regulations. charges because the Timber License authority to collect forest charges or the authority
Agreement does not expressly provide to supervise the operation by the petitioner of the
On or about June 18, 1964, during the effectivity of the License RA 3990 was enacted by the Congress that the forest charges shall be paid to the timber concession affected by said Act.
and approved by the President, which provides for a Central Experiment Station for the University of the BIR Legislative grants must be construed strictly in
Philippines. Sec. 2 of the law provides; “the parcel of the public domain consisting of three thousand favor of the public and most strongly against the
hectares, more or less, located in the Municipality of Paete, Province of Laguna, the precise boundaries of grantee, and nothing will be included in the grant
which are stated in EO 791, Series of 1961, is hereby ceded and transferred in full ownership to the except that which is granted expressly or by clear
University of the Philippines, subject to any existing concessions, if any.” implication.
BIR- the duties incident to the measuring of forest
On the strength of RA 3990, UP demanded verbally and in writing to Hardwood that (1) Hardwood shall
products and the collection of the charges
pay the forest charges under the License Agreement to UP and (2) UP Personnel shall perform the selling
(Sec.262 of the Tax Code)
of any timber felled or cut by plaintiff within the boundaries of the Central Experiment Station.
Bureau of Forestry-collection of reforestation
The civil case was filed by petitioner Hardwood before the trial court on 28 June 1966. Petitioner fees
Hardwood (1) seeks a declaration that respondent University of the Philippines (UP) does not have the B. Having been vested with administrative The cession in full ownership of the land in
right to supervise and regulate the cutting and removal of timber and other forest products covered by the jurisdiction over and being the owner of question was expressly made “subject to any
License Agreement, ceded in full ownership to the UP by RA. 3990; (2) asks that respondents be enjoined the tract of land in question, UP acquired concession, if any”, Hardwood’s concession
from committing the acts complained of; and (3) prays that respondents be required to pay petitioner the full control and benefit of the timber and would continue until 1 February 1985;
sum of P100,000.00 as damages and costs of the suit. other resources within the area UP then would acquire full ownership and
exclusive jurisdiction to control and administer
Both parties submitted a joint stipulation of facts and prayed that the Court grant relief on the following the property only after 1 February 1985. The
ISSUES: position of UP is akin to that of a donee of a parcel
of land subject to usufruct.
1) WoN Hardwood should pay forest charges due and payable under its Timber License Agreement C. Section 3 of R.A. No. 3990 expressly
to UP or to the BIR? provides: “any incidental receipts or
2) WoN UP is entitled to supervise, through its duly appointed personnel, the logging, felling and income therefrom shall pertain to the
removal of timber within the Central Experiment Station area as described in Republic Act No. general fund of the University of the
3990, and to scale the timber thus felled and cut? Philippines.”

13
D. R.A. No. 3990, UP is duty bound to of the Philippines is concerned, all its rights as grantor of the license were effectively
operate and maintain a central experiment assigned, ceded and conveyed to UP as a consequence of the above transfer of full
station, since the law does not provide for ownership. This is further borne out by Section 3 of R.A. No. 3990 which provides, inter alia,
appropriations for such purpose, it is that “any incidental receipts or income therefrom shall pertain to the general fund of the
clearly the legislative intention that the University of the Philippines.”
establishment and maintenance thereof
must be financed by the earnings or Having been effectively segregated and removed from the public domain or from a public
income from the area forest and, in effect, converted into a registered private woodland, the authority and
E. Supervision of the License Agreement in jurisdiction of the Bureau of Forestry over it were likewise terminated.
favor of petitioner by UP was intended by
R.A. No. 3990 Hardwood, as licensee or grantee of the concession, has been given the license to cut, collect, and
F. The two government agencies affected by The rulings of the Commissioner of Internal remove timber from the area ceded and transferred to UP until 1 February 1985. However, it has
R.A. No. 3990 have issued specific Revenue and the Acting Director of the Bureau of the correlative duty and obligation to pay the forest charges, or royalties, to the new owner, the
rulings recognizing the authority of UP to Forestry are patently incorrect; moreover, said UP, at the same rate as provided for in the Agreement. The charges should not be paid anymore
collect royalties or charges and to agencies do not have the power to interpret the to the Republic of the Philippines through the Bureau of Internal Revenue because of the
supervise petitioner’s logging operations. law, which is primarily a function of the judiciary very nature of the transfer as aforestated. Consequently, even the Bureau of Internal
Revenue automatically lost its authority and jurisdiction to measure the timber cut from
the subject area and to collect forestry charges and other fees due thereon.

SC Ruling: 2. Based on the nature of the transfer, it follows then that respondent UP is entitled to supervise,
through its duly appointed personnel, the logging, felling, and removal of timber within the area
1. Hardwood shall pay forest charges to the University of the Philippines. covered by R.A. No. 3990.

When R.A. No. 3990 which established a central experiment station for the use of UP in 15. TABAO V LILAGAN
connection with its research and extension functions, the above “reserved” area was “ceded and
transferred in full ownership to the University of the Philippines subject to any existing FACT:
concessions, if any.”
On February 24, 1998, a water craft M/L Hadja, from Bongao, Tawi-tawi, was docked at the port area of
When it ceded and transferred the property to UP, the Republic of the Philippines completely Tacloban City with a load of 100 tons of tanbark. Robert Hernandez was the consignee to said cargo. While
removed it from the public domain and, more specifically, in respect to the areas covered by the cargo was being unloaded, the NBI decided to verify the shipment's accompanying documents where
the timber license of petitioner, removed and segregated it from a public forest; it divested it was found to be irregular and incomplete. Consequently, the NBI ordered the unloading of the cargo
itself of its rights and title thereto and relinquished and conveyed the same to UP; and made stopped. As a result, the tanbark, the boat, and three cargo trucks were seized and impounded.
the latter the absolute owner thereof, subject only to the existing concession. On March 5, 1998, NBI-EVRO 8 Regional Director Carlos S. Caabay filed a Criminal Complaint for the
violation of Section 68 (now Section 78) of P.D. 705, The Forestry Code of the Philippines as amended,
That the law intended a transfer of the absolute ownership is unequivocally evidenced by its use against the captain and crew of the M/L Hadja, Robert Hernandez, Tandico Chion, Alejandro K. Bautista,
of the word “full” to describe it. Full means entire, complete, or possessing all particulars, or not a forster, and Marcial A. Dalimot, a Community Environment and Natural Resources Officer of the DENR.
wanting in any essential quality. The proviso regarding existing concessions refers to the timber Bautista and Dalimot were also charged with violation of Section 3(e) of R.A. No. 3019 or the Anti-Graft
license of petitioner. All that it means, however, is that the right of petitioner as a timber licensee and Corrupt Practices Act, along with Habi A. Alih and Khonrad V. Mohammad of the CENRO-Bongao,
must not be affected, impaired or diminished; it must be respected. But, insofar as the Republic Tawi-tawi. The complaint was docketed as I.S. No. 98-296 at the Prosecutor's Office of Tacloban City.
14
On March 10, 1998, DENR took possession of the cargo, the boat and the three trucks, through the previous that the shipment documents were not in order. Respondent sheriff contends that it was his ministerial duty
direction of the complainant. Due notice were issued to the consignee, Robert Hernandez and the NBI to serve the writ of replevin, absent any instruction to the contrary.
Regional Director.
The Office of the Court Administrator, in a report dated April 8, 1999, recommended that the judge be
On March 11, 1998, Hernandez filed in the RTC of Leyte a case for replevin to recover the items seized
fined in the amount of P15,000.00 for gross ignorance of the law and that the charges against respondent
by the DENR and was docketed as Civil Case No. 98-03-42.
sheriff be dismissed for lack of merit.
On March 16, 1998, subpoenas were issued to the respondents in I.S. No. 98-296 and on March 17, 1998,
ISSUE:
confiscation proceedings were conducted by the PENRO-Leyte, with both Hernandez and his counsel
present. Whether or not the respondent judge was grossly ignorant of the law and jurisprudence for issuing the writ
of replevin.
On March 19, 1998, herein respondent Judge Frisco T. Lilagan issued a writ of replevin and directed
Sheriff IV Leonardo V. Aguilar to take possession of the items seized by the DENR and to deliver them RULING:
to Hernandez after the expiration of five days. Respondent Sheriff served a copy of the writ to the
The complaint for replevin states that the shipment of tanbark and the vessel on which it was loaded were
Philippine Coast Guard station in Tacloban City at around 5:45 p.m. of March 19, 1998.
seized by the NBI for verification of supporting documents. It also stated that the NBI turned over the
Thus, the filing of this Administrative complaint against respondent via a letter addressed to the Chief seized items to the DENR "for official disposition and appropriate action". These allegations would have
Justice and dated April 13, 1998, by Atty. Tabao. been sufficient to alert the respondent judge that the DENR had custody of the seized items and that
administrative proceedings may have already been commenced concerning the shipment.
Complainant avers that replevin is not available when properties sought to be recovered are involved in
criminal proceedings. He also submits that respondent judge is either grossly ignorant of the law and Under the doctrine of primary jurisdiction, the courts cannot take cognizance of cases pending before
jurisprudence or purposely disregarded them. administrative agencies of special competence. Also, the plaintiff in the replevin suit who seeks to recover
Complainant states that the respondent sheriff had the duty to safeguard M/L Hadja and to prevent it from the shipment from the DENR had not exhausted the administrative remedies available to him. Prudent
leaving the port of Tacloban City, after he had served a writ of seizure therefor on the Philippine Coast thing for the respondent judge to do was to dismiss the replevin outright.
Guard. According to the complainant, on March 19, 1998, the vessel left the port of Tacloban City, either
Under Section 78-A of the Revised Forestry Code, the DENR secretary or his representatives may order
through respondent sheriff's gross negligence or his direct connivance with interested parties. Moreover,
the confiscation of forest products illegally cut, gathered, removed, possessed or abandoned, including the
complainant pointed out that respondent sheriff released the seized tanbark to Hernandez within the five
conveyances involved in the offense.
day period that he was supposed to keep it under the terms of the writ, thereby effectively altering,
suppressing, concealing or destroying the integrity of said evidence. It was declared by the Court in Paat vs. Court of Appeals the that enforcement of forestry laws, rules and
Respondent judge claim that the charge of gross ignorance of the law was premature since there is a regulations and the protection, development and management of forest lands fall within the primary and
pending motion to dismiss filed by the defendants in the replevin case. Further, he claimed that he was special responsibilities of the DENR. The DENR should be given free hand unperturbed by judicial
unaware of the existence of I.S. No. 98-296 and upon learning of the same, he issued an order dated March intrusion to determine a controversy which is well within its jurisdiction. The court held that the
25, 1998, suspending the transfer to Hernandez of possession of the subject items, pending resolution of assumption of the trial court of the replevin suit constitutes an unjustified encroachment into the domain
an urgent manifestation by the complainant. Respondent judges stresses that the writ of replevin was issued of the administrative ageny's prerogative. The doctrine of primary jurisdiction does not warrant a court to
in strict compliance with the requirements laid down in Rule 60 of the Revised Rule of Court. He also arrogate unto itself the authority to resolve a controversy the jurisdiction over which is initially lodged
pointed out that no apprehension report was issued by the NBI regarding the shipment and neither did the within an administrative body of special competence.
DENR issue a seizure report.
The respondent judge's act of taking cognizance of the subject replevin suit clearly demonstrates ignorance
Respondent sheriff submits that he served the writ of replevin on the Coast Guard to prevent the departure of the law. He has fallen short of the standard set forth in Canon 1 Rule 1.01 of the Code of Judicial
of subject vessel since he does not have the means to physically prevent the vessel from sailing. He further Conduct, that a judge must be an embodiment of competence, integrity and independence. To measure up
claimed that he verified the status of the cargo with DENR and that it came from a legitimate source except to this standard, justices are expected to keep abreast of all laws and prevailing jurisprudence. Failure to

15
follow basic legal commands constitutes gross ignorance of the law from which no one may be excused, of Section 3(e) of R.A. No. 3019 or the Anti-Graft and Corrupt Practices Act, 2 along with Habi A. Alih
not even a judge. and Khonrad V. Mohammad of the CENRO-Bongao, Tawi-tawi. The complaint was docketed as I.S. No.
98-296 at the Prosecutors Office of Tacloban City.
On the charges against respondent sheriff, the Court agreed with the OCA that they should be dismissed.
Respondent sheriff merely complied with his material duty to serve the writ with reasonable celerity and
In an order dated March 6, 1998, 3 complainant directed the seizure by the DENR of the M/L Hadija, its
to execute it promptly in accordance with the mandates.
cargo, and the three trucks pending preliminary investigation of the case. DENR thus took possession of
Respondent Judge Frisco T. Lilagan was found liable for gross ignorance of the law and is accordingly the aforesaid items on March 10, 1998, with notice to the consignee Robert Hernandez and the NBI
ordered to pay a fine of 10,000. 00, with a warning that a repetition of the same or similar offense will be Regional Director.
dealt more severely. The complaint against respondent Sheriff IV Leonardo V. Aguilar is dismissed for
lack of merit. On March 11, 1998, Hernandez filed in the Regional Trial Court of Leyte a case for replevin to recover
the items seized by the DENR. The case was raffled off to Branch 34 of said court and docketed as Civil
FULL CASE Case No. 98-03-42.

This is an administrative complaint filed by Atty. Leo C. Tabao, Assistant City Prosecutor of Tacloban, in On March 16, 1998, subpoenas were issued to the respondents in I.S. No. 98-296. On March 17, 1998,
his capacity as Regional Chairman of the Region 8 Special Task Force on Environment and Natural confiscation proceedings were conducted by the Provincial Environment and Natural Resources Office
Resources, against (1) Judge Frisco T. Lilagan, presiding judge of the Leyte Regional Trial Court, Branch (PENRO)-Leyte, with both Hernandez and his counsel present.
34, for gross ignorance of the law, gross abuse of judicial authority, and willful disobedience to settled
jurisprudence; and (2) Sheriff IV Leonardo V. Aguilar of the Leyte RTC, Office of the Clerk of Court, for On March 19, 1998, herein respondent Judge Frisco T. Lilagan issued a writ of replevin and directed
gross irregularity in the performance of official duties, giving unwarranted benefits to a private individual, respondent Sheriff IV Leonardo V. Aguilar to take possession of the items seized by the DENR and to
violation of Section 1(b) and (c) of P.D. No. 1829, and conduct prejudicial to the best interest of the service. deliver them to Hernandez after the expiration of five days. 4 Respondent sheriff served a copy of the writ
to the Philippine Coast Guard station in Tacloban City at around 5:45 p.m. of March 19, 1998.
The records of this case reveal the following facts.
Thus, the filing of this administrative complaint against respondents via a letter addressed to the Chief
On February 24, 1998, a water craft registered under the name M/L Hadija, from Bongao, Tawi-tawi, was Justice and dated April 13, 1998, by Atty. Tabao.
docked at the port area of Tacloban City with a load of around 100 tons of tanbark. Due to previous
irregular and illegal shipments of tanbark from Bongao, agents of the National Bureau of Investigation in Complainant avers that replevin is not available where the properties sought to be recovered are involved
Region 8 (NBI-EVRO #8) decided to verify the shipments accompanying documents as the M/L Hadija in criminal proceedings for illegal logging. He points out that this is a well-settled issue and cites several
was unloading its cargo to its consignee, a certain Robert Hernandez. decisions 5 of this Court and the Court of Appeals on the matter. He argues that respondent judge should
have known of the existing jurisprudence on this issue, particularly since they are subject to mandatory
The NBI agents found the documents irregular and incomplete, and consequently they ordered the judicial notice per Section 1, Rule 129 of the Revised Rules of Court.
unloading of the cargo stopped. The tanbark, the boat M/L Hadija, and three cargo trucks were seized and
impounded. Complainant submits that respondent judge is either grossly ignorant of the law and jurisprudence or
purposely disregarded them. But he avers that it is respondent judges duty to keep abreast of developments
On March 5, 1998, NBI-EVRO #8 Regional Director Carlos S. Caabay filed a criminal complaint for in law and jurisprudence.
violation of Section 68 (now Section 78) of P.D. No. 705, 1 the Forestry Reform Code of the Philippines
(as amended), against the captain and crew of the M/L Hadija, Robert Hernandez, Tandico Chion, Complainant claims that respondent judge cannot claim ignorance of the proceedings in I.S. No. 98-296
Alejandro K. Bautista, and Marcial A. Dalimot. Bautista was a forester while Dalimot was a Community for the following reasons: (1) the defendants in the replevin case were all DENR officers, which should
Environment and Natural Resources Officer (CENRO) of the Department of Environment and Natural have alerted respondent judge to the possibility that the items sought to be recovered were being held by
Resources (DENR) office in Tacloban City. Bautista and Dalimot were, thus, also charged with violation the defendants in their official capacities; and (2) the complaint for replevin itself states that the items were
16
intercepted by the NBI for verification of supporting documents, which should have made respondent Respondent judge stresses that the writ of replevin was issued in strict compliance with the requirements
judge suspect that the same were being held by authority of law. laid down in Rule 60 of the Revised Rules of Court. He also points out that said writ was issued
provisionally and was not intended to be the final disposition of the replevin case.
As regards respondent sheriff Leonardo V. Aguilar, complainant states that it was incumbent upon Aguilar
to safeguard the M/L Hadija and prevent it from leaving the port of Tacloban City, after he had served a Respondent judge avers that the charge of gross ignorance of the law is premature since he has not made
writ of seizure therefor on the Philippine Coast Guard. However, on March 19, 1998, the vessel left the a ruling yet on the motion to dismiss filed in the replevin case. He contends that it was too much to ask
port of Tacloban City, either through respondent sheriffs gross negligence or his direct connivance with from him to take note of the fact that the defendants in said case were officials of DENR and make
interested parties, according to complainant. As of the time of the filing of the complaint, according to assumptions based on such fact. Moreover, respondent judge submits that while the complaint alleged that
complainant, the whereabouts of the vessel and its crew were unknown. the cargo of tanbark was intercepted by the NBI, it also alleged that the consignee thereof produced
documents to prove that the shipment was legal.
Moreover, complainant points out that respondent sheriff released the seized tanbark to Hernandez on
March 20 and 21, 1998, or within the five-day period that he was supposed to keep it under the terms of In conclusion, respondent judge points out that no apprehension report was issued by the NBI regarding
the writ. Complainant argues that the tanbark formed part of the peoples evidence in the criminal complaint the shipment. Neither did the DENR issue a seizure report. Respondent judge contends that the validity of
against Hernandez and the others. By his act, respondent sheriff effectively altered, suppressed, concealed, the seizure of the subject items by the DENR is a matter that will have to be resolved in relation to the
or destroyed the integrity of said evidence. For this act, complainant contends that respondent sheriff may motion to dismiss.
be held liable under Section 1(b) of P.D. 1829, Penalizing Obstruction of Apprehension and Prosecution
of Criminal Offenders. 6 Respondent sheriffs acts also constitute gross irregularity in the performance of For his part, respondent sheriff submits 8 that he served the writ of replevin on the Coast Guard precisely
his duty as a court employee. to prevent the departure of the subject vessel, since he does not have the means to physically prevent said
vessel from sailing. The Coast Guard commander should have examined the vessel and its crew after being
Complainant notes that respondent sheriff was absent from his office from March 20 to March 24, 1998. served the writ, to determine whether or not they were engaged in any illegal activity.
This period included the dates he was supposed to have released the tanbark to Hernandez. Complainant
contends that respondent sheriff not only unlawfully released the tanbark, he also made it appear that he Respondent sheriff narrates that no cargo was on board the vessel when he served the writ on the Coast
was not physically present when such act was done. Guard. He verified the cargos status with DENR, which furnished him a copy of a fax transmission stating
that the tanbark came from legitimate sources except that the shipment documents were not in
In separate indorsements dated September 9, 1998, then Court Administrator Alfredo L. Benipayo referred order. 9cräläwvirtualibräry
this administrative matter to both respondents for comment.
Respondent sheriff contends that it was his ministerial duty to serve the writ of replevin, absent any
In his comment dated October 12, 1998, 7 respondent judge calls the attention of the Office of the Court instruction to the contrary. He argues further that since the items subject of the writ are in the custody of
Administrator to a pending motion to dismiss filed by the defendants in the replevin case that effectively the court and could be disposed of only through court order, there could not be any unwarranted benefit to
prevented him from commenting on the issue. The discussions that would have to be included in the a private individual as claimed by complainant.
comment, he says, would also resolve the pending motion to dismiss. Respondent judge contends that
complainant should have been prudent enough to wait for the resolution of the motion to dismiss before Noting that the questioned shipment of tanbark was not covered by either an NBI apprehension report or
filing the instant administrative case. a DENR seizure report, respondent sheriff contends that complainant should have taken steps to protect
the integrity of the shipment instead of heaping blame upon others for his own negligence. Respondent
Respondent judge claims that he was unaware of the existence of I.S. No. 98-296. He only learned of the sheriff avers that it was not his intention to obstruct the apprehension and prosecution of criminal
criminal case from an urgent manifestation dated March 20, 1998, filed by complainant. He argues that he offenders, contrary to complainants claim.
issued an order dated March 25, 1998, suspending the transfer to Hernandez of possession of the subject
items, pending resolution of the urgent manifestation.

17
Respondent sheriff refutes complainants claim that he was absent from his office from March 20 to March Natural Resources. By the very nature of its function, the DENR should be given a free hand unperturbed
24, 1998, and alleges that it was complainant who was absent from court hearings on several occasions, in by judicial intrusion to determine a controversy which is well within its jurisdiction. The assumption by
violation of his duty as a prosecutor. the trial court, therefore, of the replevin suit filed by private respondents constitutes an unjustified
encroachment into the domain of the administrative agencys prerogative. The doctrine of primary
Respondent submitted two supplemental comments dated October 30, 1998, 10 and May 3, 1999, 11 (1) jurisdiction does not warrant a court to arrogate unto itself the authority to resolve a controversy the
reiterating his contention that the tanbark seized by the DENR and subject of the replevin case had been jurisdiction over which is initially lodged with an administrative body of special competence.
found to come from a legitimate source, per an order signed by the Regional Director (Region 8) of the xxx19cräläwvirtualibräry
DENR, 12 and (2) informing the OCA that the main replevin case was dismissed per an order of respondent
judge dated November 27, 1998. 13cräläwvirtualibräry Respondent judges act of taking cognizance of the subject replevin suit clearly demonstrates ignorance of
the law. He has fallen short of the standard set forth in Canon 1, Rule 1.01 of the Code of Judicial Conduct,
As required by resolution of the Court dated January 24, 2001, the parties herein separately manifested that a judge must be the embodiment of competence, integrity, and independence. To measure up to this
that they are willing to have the present case resolved based on the record on hand. standard, judges are expected to keep abreast of all laws and prevailing jurisprudence. 20 Judges are duty
bound to have more than just a cursory acquaintance with laws and jurisprudence. Failure to follow basic
We note that in its report dated April 8, 1999, the OCA, after reviewing the case, recommended that legal commands constitutes gross ignorance of the law from which no one may be excused, not even a
respondent judge be fined in the amount of P15,000.00 for gross ignorance of the law. At the same time, judge. 21cräläwvirtualibräry
the OCA recommended that the charges against respondent sheriff be dismissed for lack of merit.
We find, however, that respondent judge had already vacated the Writ of Seizure he issued on March 19,
The recommendation of the OCA is well taken, except for the amount of the fine to be imposed on said 1998, in a subsequent Order dated November 27, 1998, dismissing the Civil Complaint for replevin filed
respondent judge. by Robert Hernandez against the Regional Director of the DENR and other officers. He also directed in
said order the sheriff to return to CENRO, Tacloban City, all the chattels confiscated by virtue of the Writ
The complaint for replevin itself states that the shipment of tanbark as well as the vessel on which it was of Seizure.22cräläwvirtualibräry
loaded were seized by the NBI for verification of supporting documents. 14 It also states that the NBI turned
over the seized items to the DENR for official disposition and appropriate action. 15 A copy of the Further, we find that Sheriff Aguilar in his Final Return of the Writ, dated December 15, 1998, had already
document evidencing the turnover to DENR was attached to the complaint as Annex D. 16 To our mind, delivered to CENRO the 102 tons and 120 kilos of tanbark duly receipted by CENRO representative
these allegations would have been sufficient to alert respondent judge that the DENR has custody of the Marcial A. Dalimot on the same date. 23cräläwvirtualibräry
seized items and that administrative proceedings may have already been commenced concerning the
shipment. Under the doctrine of primary jurisdiction, courts cannot take cognizance of cases pending The OCA recommends that respondent judge be fined in the amount of P15,000.00. Under the
before administrative agencies of special competence. 17 Note, too, that the plaintiff in the replevin suit circumstances, considering that this is the first complaint against him, we deem a fine of P10,000.00 to be
who seeks to recover the shipment from the DENR had not exhausted the administrative remedies available sufficient.
to him. 18 The prudent thing for respondent judge to have done was to dismiss the replevin suit outright.
Regarding the charges against respondent sheriff, we agree with the OCA that they should be dismissed.
Under Section 78-A of the Revised Forestry Code, the DENR secretary or his authorized representatives Respondent sheriff merely complied with his ministerial duty to serve the writ with reasonable celerity
may order the confiscation of forest products illegally cut, gathered, removed, or possessed or abandoned, and to execute it promptly in accordance with its mandates. 24cräläwvirtualibräry
including the conveyances used in the commission of the offense.
WHEREFORE , respondent Judge Frisco T. Lilagan is hereby found liable for gross ignorance of the law
In this regard, we declared in Paat v. Court of Appeals: and is accordingly ordered to pay a FINE of P10,000.00, with a WARNING that a repetition of the same
or a similar offense will be dealt with more severely. The complaint against respondent Sheriff IV
the enforcement of forestry laws, rules and regulations and the protection, development and management Leonardo V. Aguilar is DISMISSED for lack of merit.
of forest lands fall within the primary and special responsibilities of the Department of Environment and
18
16. TAN VS DIR OF FORESTRY 2. No. Petitioner has not exhausted all his administrative remedies. Petitioner did not appeal the order of the respondent Secretary
of Agriculture and Natural Resources to the President of the Philippines who issued Executive Proclamation No. 238
withdrawing the area from private exploitation, and establishing it as the Olongapo Watershed Forest Reserve. Considering the
President has the power to review on appeal the orders or acts of the respondents, the failure of the petitioner to take that appeal is
Facts: failure on his part to exhaust his administrative remedies.
On April 15, 1963, petitioner-appellant Wenceslao Vinzon Tan won the bididing for the license of logging operations on a
public forest land situated in Olongapo, Zambales. Secretary of Agriculture and Natural Resources Benjamin Gozon issued 3. Yes. Petitioner’s action is a suit against the State.
General Memorandum, Order No. 46 which states that “The Director of Forestry is hereby authorized to grant a new ordinary Under the doctrine of State immunity, a suit against the State cannot prosper unless the State has given its consent. Both the
timber licenses where the area covered thereby is not more than 3,000 hectares each and that the extension of an ordinary timber Secretary of Agriculture and Natural Resources and Director of Forestry acted in their capacity as officers of the State and
license for areas not exceeding 5,000 hectares.” representatives of the sovereign authority discharging governmental powers. Petitioner’s action is an attempt to circumvent the
Thereafter Jose Y. Feliciano was appointed as Acting Secretary of Agriculture and Natural Resources. Upon assumption of rule establishing State exemption from suit. The promotion of public welfare and the protection of the inhabitants near the public
office on December 19,1963 he promulgated General Memorandum Order No. 46 which took effect on the same day. Such forest are property, rights and interest of the State. Accordingly, it may not be circumvented by directing the action against the
General Memorandum is a revocation of General Memorandum No. 46 which states that “Issuance of new licenses, including officer of the State instead of against the State itself.
amendments shall be signed by the secretary of Agriculture and Natural Resources.” On the same day the Acting Director of
Forestry signed the timber license of the petitioner without the approval of the Secretary of Agriculture and Natural Resources 4.Yes. The revocation of the petitioner’s timber license by the respondent is a valid exercise of his power as Secretary of
and was released by the Office of the Director of Forestry on January 6, 1964.On February 19, 1964. Ravago Commercial wrote Agriculture and Natural Resources. The Director of Forestry who issued the timber license of the petitioner is under the
a letter to the Secretary of Agriculture and Natural Resources praying that the license issued to petitioner be cancelled or revoked supervision of the Head or Secretary of the Agriculture and Natural Resources. The power and control of the Department Heads
on the ground that the grant was irregular, anomalous and contrary to the existing forest rules, laws and regulations. On March 9, over bureaus and offices includes the power to modify, reverse or set aside acts of subordinate officials. Accordingly, respondent
1964, Feliciano promulgated an order declaring Ordinary Timber License NO. 20-64 issued to petitioner by the Director of as Secretary of Agriculture and Natural Resources has the authority to revoke, on valid grounds the timber license issued by the
Forestry without authority is void ab initio. Petitioner moved for reconsideration of the order but Feliciano denied the motion. On Director of Forestry.
the basis of the denial for his motion to reconsideration, petitioner filed the instant case before the court a quo, a petition for
certiorari prohibiting and mandamus with preliminary prohibitory injunction. Petitioner claims that respondent unlawfully, 5.Yes. Petitioner’s timber license can still be revoked even if it is valid.
illegally and arbitrarily acted in excess of their jurisdiction with grave abuse of discretion by revoking his license Paragraph 27 of the rules and regulations included in the ordinary timber license states that the terms and conditions of the
without a valid cause, by denying him of equal protection of law, by depriving him of his constitutional right to property license are subject to change at the discretion of the Director of Forestry. A timber license is not a contract within the purview of
without due process of law and by impairing the obligation of contract. The court a quo, from the evidence received, declared the due process of law but a privilege which can be withdrawn whenever dictated by the public interest or welfare.
that the petition did not state a sufficient cause of action and dismissed the same. Petitioner, having his motion for reconsideration
denied, appealed directly to the Supreme Court.
Issue:
1.Whether or not the timber license issued to the petitioner is void abinitio. 17. PEOPLE VS. QUE
2.Whether or not petitioner has exhausted all his administrativeremedies.
3.Whether or not petitioner’s action is a suit against the State. Facts
4.Whether or not the revocation of the petitioner’s timber license by the respondent is a valid exercise of his power as Secretary • SPO1 Corpus, a member of the Provincial Task Force on Illegal Logging received an info that a 10-
of Agriculture and Natural Resources. wheeler truck had illegally obtained lumbers. So together with his team went on a patrol. They followed
5.Whether or not petitioner’s timber license can still be revoked even if it is valid. the truck and apprehended it
• 3 persons, including Que were inside the truck. The truck contained coconut slabs qith sawn lumbers in
Ruling:
1.Yes. The timber license issued to the petitioner is void ab initio. between them
Petitioner’s timber license was signed and released without the • Corpuz asked for the necessary evidences to prove the legality of the origin of the materials. But Que
authority of the by then Acting Director Estanislao Bernal of Forestry. only showed a certification from CENRO (Community Environmental and Natural Resources Office
• Que was charges in RTC with violation og Sec 68 of PD 705 as amended by EO 277
At the time it was released, the Acting Director of Forestry had no more authority to grant a license. The authority • Trial court found Que guilty and sentenced him to reclusion perpetua and also confiscated the seized
delegated to him was contained in General Memorandum Order No. 46 which was revoked by General Order No. 60 which lumbers
removes the authority of the Director of Forestry to grant license.

Issue

19
• Whether there are no existing forest laws and regulations which required certain legal documents for Office in Quezon City. There, petitioner Atty. Vicente Robles of the PIC/SAID investigated them, and
possession of timber and other forest products. discovered the discrepancies in the documentation of the narra lumber.
• Whether the law only penalizes possession of illegal forest products and that the possessor cannot be
What were declared in the documents were narra flitches, while the cargo of the truck consisted of narra
held liable if he proves that the cutting, gathering, collecting or removal of such forest products is legal
lumber. In the documents, the plate numbers of the truck supposed to carry the cargo bear the numbers
BAX-404, PEC-492 or NSN-267, while the plate of the truck apprehended is NVT-881. Considering that
Held
the cargo is lumber, the transport should have been accompanied by a Certificate of Lumber Origin, scale
• The interpretation of the appellant with “existing forest laws and regulations” refer to laws and
sheet of said lumber and not by a Certificate of Timber Origin. The Log Sale Purchase Agreement
regulations” to refer to those laws and regulations which were already in effect at the time of enactment of
presented is between DSM Golden Cup International as the Seller and Bonamy Enterprises as the
EO 277, would be strained and would render the law inutile. The phrase should be construed to refer to
buyer/consignee and not with Lily Francisco Lumber Hardware.
laws and regulations existing at the time of possession of timber or other forest products. Section 3 of the
Administrative Order provides that the movement of logs, lumber, non-timber forest products and These are in violation of Bureau of Forestry Development (BFD) Circular No. 10 which requires
woodbased or wood based shall be covered with the appropriate Certificates of Origin. The transport of possession or transportation of lumber to be supported by the following documents:
lumber shall be accompanied by CLO (Certificate of Lumber Origin). o Therefore, the accused was given
permit by DENR to transport one (1) truckload of coconut slabs only between March 7 to 11, 1994. The 1. Certificate of Lumber Origin (CLO) which shall be issued only by the District
accused was apprehended on March 8, 1994 aboard his truck which was loaded not only with coconut Forester, or in his absence, the Assistant District Forester;
slabs but with chainsawn lumber as well. Admittedly, the lumber could not be seen from the outside. The 2. Sales Invoice;
lumber were placed in the middle and not visible unless the coconut slabs which were placed on the top, 3. Delivery Receipt; and
sides and rear of the truck were removed
4. Tally Sheets.
• No, because there are 2 distinct and separate offenses punished under Section 68 of P.D. 705 o There are
2 distinct and separate offenses punished under Sec 68 of PD 705 § Cutting, gathering, collecting and Such omission is punishable under Sec. 68 of Presidential Decree (P.D.) No. 705 otherwise known as the
removing timber or other forest products from any forest land, or timber from alienable or disposable Revised Forestry Code. Thus, petitioner Atty. Robles issued a temporary seizure order and seizure receipt
public land, or from private land without any authority; and § Possession of timber or other forest products for the narra lumber and the six-wheeler truck.
without the legal documents required under existing forest laws and regulations. o In the first offense, one On January 20, 1989, petitioner Fulgencio S. Factoran, then Secretary of Environment and Natural
can raise as a defense the legality of the acts of cutting, gathering, collecting, or removing timber or other Resources issued an order for the confiscation of the narra lumber and the six-wheeler truck
forest products by presenting the authorization issued by DENR. Second, s=t is immaterial whether the
cutting, gathering, collecting and removal of the forest products is legal or not. Mere possession of forest Private respondents neither asked for reconsideration of nor appealed the said order to the Office of the
products without the proper documents consummates the crime. Whether or not the lumber comes from a President. Consequently, the narra lumber and six-wheeler truck were forfeited in favor of the government
legal source is immaterial because E.O. 277 considers the mere possession of timber or other forest and were later on advertised to be sold at a public auction on March 20, 1989.
products without the proper legal documents as malum prohibitum.
On March 17, 1989, private respondents filed a complaint with prayer for the issuance of the writs of
replevin and preliminary injunction and/or temporary restraining order for the recovery of the confiscated
items, and to enjoin the panned auction sale of the subject narra lumber, respectively.
18. FACTORAN VS CA
On the same day, the trial court issued an order directing the parties to desist from proceeding with the
FACTS: planned auction sale and setting the hearing for the issuance of the writ of preliminary injunction on March
27, 1989.
On August 9, 1988 two police officers of Marikina Police Station, Sub-Station III, intercepted a six-
wheeler truck carrying 4,000 board feet of narra lumber as it was cruising along Marcos Highway. They On March 20, 1989, private respondents filed and Ex-Parte motion for Release and Return of Goods and
apprehended the truck driver, private respondent Jesus Sy, and brought the truck and its cargo to the Documents (Replevin) supported by an Affidavit for Issuance of Writ of Replevin and Preliminary
Personnel Investigation Committee/Special Actions and Investigation Division (PIC/SAID) of DENR Injunction and a Replevin Bond in the amount of P180,000.00. The trial court granted the writ of replevin

20
on the same day and directed the petitioners "to deliver the xxx [n]arra lumber, original documents and Whether or not the RTC was correct in the issuance of a writ of replevin and the Court of Appeals in
truck with plate no. NJT 881 to the custody of the plaintiffs and/or their representatives x x x". dismissing the petition and lifting the preliminary injunction.

On March 22, 1989, the trial court issued a writ of seizure. However, petitioners refused to comply RULING:
therewith. Sheriff David G. Brodett of Branch 80 of the RTC of Quezon City, reported that the petitioners
Pursuant to Sec. 8 of P.D. No. 705, all actions and decision of the Director are subject to review, motu
prevented him from removing the subject properties from the DENR compound and transferring them to
propio or upon appeal of any person aggrieved thereby, by the Department Head whose decision shall be
the Mobile Unit compound of the Quezon City Police Force. He then agreed to a constructive possession
final and executory after the lapse of 30 days from the receipt by the aggrieved party of said decision
of the properties. On that same day, petitioners filed a Manifestation stating their intention to file a
unless appealed to the President. The decision of the Department Head may not be reviewed by the courts
counterbond under Rule 60 of the Rules of Court to stay the execution of the writ of seizure and to post a
except through a special civil action for certiorari or prohibition.
cash bond in the amount of P180,000.00. The trial court did not oblige the petitioners for they failed to
serve a copy of the Manifestation on the private respondents. Petitioners then made immediately the It was observed by the Court that herein respondents never appealed the confiscation order of the petitioner
required service and tendered the cash counterbond but it was refused, petitioners' Manifestation having Secretary to the Office of the President.
already been set for hearing on March 30, 1989.
The doctrine of exhaustion of administrative remedies is basic. Courts, for reasons of law, comity and
On March 27, 1989, petitioners made another attempt to post a counterbond but was also denied for the convenience, should not entertain suits unless the available administrative remedies have first been
same reason. resorted to and proper authorities have been given an appropriate opportunity to act and correct their
alleged errors, if any, committed in the administrative forum.
On the same day, private respondents filed a motion to declare petitioners in contempt for disobeying the
writ of seizure. The trial court gave petitioners 24 hours to answer the motion. Hearing was scheduled on It was pointed out by the Court in Paat vs. Court of Appeals that the enforcement of forestry laws, rules
March 30, 1989. and regulations and the protection, development and management of forest land fall within the primary
and special responsibilities of the DENR. It held that assumption of the trial court of a replevin suit
On March 29, 1989, petitioners filed with the Court of Appeals a Petition for Certiorari, Prohibition and/or
constitutes an encroachment into the domain of the administrative agency's prerogative. The doctrine of
Mandamus to annul the orders of the trial court dated March 20, 1989 and March 27, 1989.
preliminary jurisdiction does not warrant a court to arrogate unto itself the authority to resolve a
On March 30, 1989, the Court of Appeals granted petitioners temporary relief in the form of a temporary controversy the jurisdiction over which is initially lodged with an administrative body of special
restraining order (TRO). competence.

On September 11, 1989, the Court of Appeals converted the TRO into a writ of preliminary injunction However, herein petitioners did not a motion to dismiss on the ground of non-exhaustion of administrative
upon filing by petitioners of a bond in the amount of P180,000.00. remedies. Thus, it is deemed waived.

On March 30, 1990, the Court of Appeals lifted the writ of preliminary injunction and dismissed the Nonetheless, the Court finds the petition impressed with merit.
petition. It declared that the complaint for replevin filed by the private respondents complied with the
First. A writ of replevin does not issue as a matter of course upon the applicant's filing of a bond and
requirements of an affidavit and bond under Sec. 1 and 2 of Rule 60 of the Revised Rules of Court, issuance
affidavit, as the Court of Appeals has wrongly put it. The mere filing of an affidavit, sans allegations
of the writ of replevin was mandatory.
therein that satisfy the requirements of Section 2 Rule 60 of the Revised Rules of Court, cannot justify the
As for the contempt charges against the petitioners, the Court of Appeals believed that the same were issuance of a writ of replevin. Wrongful detention of the properties sought in an action for replevin must
sufficiently based on a written charge by private respondents and the reports submitted by the Sheriff. be satisfactory established. If only mechanistic averment thereof is offered, the writ should not be issued.

On April 25, 1990, petitioners filed a motion for reconsideration of the foregoing decision but it was In the case at bar, the taking of the subject property was within the administrative authority of the Secretary
subsequently denied by the Court of Appeals in its Resolution dated May 18, 1990. as provided by Section 68-A of P.D. No. 705. Thus, it is not wrongful and does not warrant the issuance
of a writ of replevin prayed for by the private respondents.
Hence this petition.

ISSUE:
21
Second. By virtue of the confiscation order by petitioner Secretary, the subject properties of private because the petitioner failed to produce upon demand the required legal documents to prove the legitimacy
respondents were held in custodia legis and hence, beyond the reach of replevin. Property lawfully taken of their source and origin.
by virtue of legal process is deemed to be in custodia legis. So basic is this doctrine that it found inclusion
in the 1997 amendments introduced to the Rules of Civil Procedure. Sec. Factoran ordered the suspension of Mustang Lumber lumber-dealer permit. He also ordered to
confiscate in favor of the government to be disposed of in accordance with law the seized lumber inside
Third. Petitioner Secretary's authority to confiscate forest products under SEction 68-A of P.D. No. 705 is
petitioner’s lumberyard.
distinct and independent of the confiscation of forest products in a criminal action provided for in Section
68 of P.D. No. 705.
Mustang Lumber filed a petition for certiorari and prohibition against DENR Sec and SAID Chief with
Fourth. SEction 80 of P.D. No. 705 which requires the delivery of the seized forest products within six (6) the RTC questioning the seizure without any search and seizure order from the judge and the orders of the
hours from the time of the seizure to the appropriate official designated by law to conduct preliminary DENR Sec. for lack of prior notice and hearing (due process).
investigations applies only to criminal prosecutions provided for in Section 68 and not to administrative
confiscation provided for in Section 68-A. Trial Court decision (Civil Case): The warrantless search and seizure of the petitioner's truck was valid.
Search of a moving vehicle is one of the exceptions where warrantless search and seizure is justified. The
Fifth. Nothing in the records supports private respondents' allegation that their right to due process was seizure of large volume of lumber was a continuation of that made the previous day and was still pursuant
violated as no investigation was conducted prior to confiscation of their properties. to or by virtue of the search warrant issued by Judge Osorio whose validity the petitioner did not even
question. Even if the search warrant did not specifically mention almaciga, supa, and lauan lumber and
Finally. The writ or seizure and the writ of replevin was issued by the trial court in grave abuse of its
shorts, their seizure was valid because it was contraband or prohibited articles.
discretion. Thus, disobedience thereto cannot constitute indirect contempt of court which presupposes that
the court order thereby violated was valid and legal. Without a lawful order being issued, no contempt of
Chief Robles filed with the DOJ a complaint against Ri Chuy Po, Mustang Lumber’s President and General
court could be committed.
Manager. An Information was filed by DOJ with the RTC charging Ri Chuy Po for violating Sec. 68 of
The instant petition is granted. The decision of the Court of Appeals dated March 30, 1990 and its PD No. 705
Resolution dated May 18, 1990 were set aside. Respondent presiding judge of the RTC of Quezon City
was permanently enjoined from enforcing the Orders dated March 20, 1989 and March 22, 1989, or if said Ri Chuy Po defense in Criminal Case: lumber, as opposed to timber, is not penalized in Sec. 68 of PD
orders had already been issued, said respondent judge was directed to render judgement of forfeiture of No. 705. Even if lumber is included, it cannot be used against him because it was seized illegally. Also,
replevin bond filed by private respondents. Finally, the said respondent judge is hereby permanently the pending case in CA regarding the legality of the seizure raises a prejudicial question.
enjoined from further acting on the Motion for Contempt filed by private respondents against petitioners.
Prosecution’s opposition: Lumber is included in Sec. 68 of PD No. 705 and possession of such without
the required documents is penalized. Also, exclusion of lumber from 68 would defeat the purpose of the
law which is to halt illegal logging
19. MUSTANG LUMBER, INC. V. CA; DENR Sec. Factoran; SAID Chief Robles | Davide, Jr., J
G.R. No. 104988 | June 18, 1996 Trial Court decision (Criminal Case): Granted the motion to quash and dismissed the case on the ground
that possession of lumber without the required documents was not a crime.
Facts: SAID organized a team of foresters and policemen to conduct surveillance in the lumberyard of People filed a petition for certiorari with the SC contending that respondent judge acted with grave abuse
Mustang Lumber. They saw the truck of petitioner loaded with lauan and almaciga lumber of assorted of discretion when she granted the motion to quash and dismissed the case.
sizes and dimensions. The team seized the truck including its cargo and impounded them at the DENR
compound in Quezon City because the driver was not able to provide the required invoices. They were not CA decision: Dismissed the appeal of Mustang Lumber for lack of merit and affirmed the trial court’s
able to enter the lumberyard because of the owner’s refusal. The team secured a search warrant from Judge ruling. They were not able to show the required legal documents for the possession of lumber.
Osorio of RTC Valenzuela. By virtue thereof, they seized truckloads of shorts, slabs and lumber from the
lumberyard of Mustang Lumber. The other remaining stockpiles were placed under administrative seizure

22
Held: The word lumber does not appear in Sec. 68 of PD No. 705 however lumber is included in the term
timber. Lumber is a processed forest raw material. The Revised Forestry Code uses the term lumber in its 20. TAOPA v. PEOPLE
common usage. Even Webster Dictionary defines lumber as a timber or logs after being prepared for the
market. Simply put lumber is a processed timber. Judge Dizon-Capulong committed grave abuse of FACTS:
discretion in dismissing the criminal case
the Community Environment and Natural Resources Office of Virac, Catanduanes seized a truck loaded
The seizure the truck and its cargo was a valid exercise of the power vested upon a forest officer or with illegally-cut lumber (113 pieces of lumber of Philippine Mahogany Group and Apitong species
employee by Section 80 of P.D. No. 705, as amended by P.D. No. 1775. Search of a moving vehicle is one without any authority and/or legal documents as required under existing forest laws and regulations,
of the five doctrinally accepted exceptions to the constitutional mandate that no search or seizure shall be prejudicial to the public interest.) and arrested its driver, Placido Cuison. The lumber was covered with
made except by virtue of a warrant issued by a judge after personally determining the existence of probable bundles of abaca fiber to prevent detection. On investigation, Cuison pointed to petitioner Amado Taopa
cause. The other exceptions are (1) search as an incident to a lawful arrest, (2) seizure of evidence in plain and a certain Rufino Ogalesco as the owners of the seized lumber.
view, (3) customs searches, and (4) consented warrantless search
Taopa, Ogalesco and Cuison were charged with violating Section 68 of Presidential Decree (PD) No. 705
They are presumably trifling attempts to block the serious efforts of the DENR to enforce the decree, as amended, in the RTC Virac, Catanduanes. Taopa, Ogalesco and Cuison pleaded not guilty on
efforts which deserve the commendation of the public in light of the urgent need to take firm and decisive arraignment. After trial on the merits, RTC found them guilty as charged beyond reasonable doubt.
action against despoilers of our forests whose continuous destruction only ensures to the generations to
come, if not the present, an inheritance of parched earth incapable of sustaining life. The Government must Only Taopa and Cuison appealed to CA, Cuison was acquitted but Taopa's conviction was affirmed.4
not tire in its vigilance to protect the environment by prosecuting without fear or favor any person who The dispositive portion of the CA decision read:
dares to violate our laws for the utilization and protection of our forests. In this petition, Taopa seeks his acquittal from the charges against him alleging that the prosecution failed
to prove that he was one of the owners of the seized lumber as he was not in the truck when the lumber
Dispositive portion: was seized.

WHEREFORE, judgment is hereby rendered ISSUE:


WON taopa is guilty of violating Section 68 of PD No. 705, as amended?
1. (a) GRANTING the petition in G.R. No. 106424; (b) SETTING ASIDE and ANNULLING, for
having been rendered with grave abuse of discretion, the challenged orders of 16 August 1991 HELD:
and 18 October 1991 of respondent Judge Teresita Dizon-Capulong, Branch 172, Regional Trial YES. Petition is denied. CA decision affirmed with modification. Petitioner Amado Taopa is hereby found
Court of Valenzuela, Metro Manila, in Criminal Case No. 324-V-91, entitled People of the GUILTY beyond reasonable doubt for violation of Section 68 of PD No. 705, as amended, and sentenced
Philippines vs. Ri Chuy Po; (c) REINSTATING the information in the said criminal case; and (d) to suffer the indeterminate penalty of imprisonment from 10 years and one day of prision mayor, as
DIRECTING the respondent Judge on her successor to hear and decide the case with purposeful minimum, to 20 years of reclusion temporal as maximum, with the accessory penalties provided for by
dispatch; and law.
2. DENYING the petitions in G.R. No. 104988 and in G. R. No. 123784 for utter failure of the
petitioner to show that the respondent Court of Appeals committed any reversible error in the Both RTC and CA gave no consideration to Taopa's alibi because Cusion's testimony proved Taopa's
challenged decisions of 29 November 1991 in CA-G.R. SP No. 25510 in the FIRST CIVIL CASE active participation in the transport of the seized lumber
and of 31 July 1995 in CA-G.R. SP No. 33778 in the SECOND CIVIL CASE.
Costs against the petitioner in each of these three cases. RTC and CA found that the truck was loaded with the cargo in front of Taopa's house and that Taopa and
Ogalesco were accompanying the truck driven by Cuison up to where the truck and lumber were seized.
SO ORDERED. These facts proved Taopa's (and Ogalesco's) exercise of dominion and control over the lumber loaded in

23
the truck. The acts of Taopa (and of his co-accused Ogalesco) constituted possession of timber or other years, five months and 11 days of reclusion temporal to 22 years, two months and 21 days of reclusion
forest products without the required legal documents. perpetua.

The mere fact that Taopa and Ogalesco ran away at the mere sight of the police was likewise largely
indicative of guilt. Court is convinced that Taopa and Ogalesco were owners of the seized lumber.
21. Galo vs. Monge, G. R. No. 170308
However, Court disagree with RTC and CA as to the penalty imposed on Taopa. Section 68 of PD 705, as Facts:
amended,7 refers to Articles 309 and 310 of the Revised Penal Code (RPC) for the penalties to be imposed
on violators. Violation of Section 68 of PD 705, as amended, is punished as qualified theft.8 On July 20, 1994, Monge(petitioner) and Potencio were found by the barangay tannods in
possession of and transporting 3 pieces of mahogany lumber in Iriga City. Monge and Potencio were not
The law treats cutting, gathering, collecting and possessing timber or other forest products without license able to show any documents or the requisite permit from DENR. The trial court found Monge guilty of
as an offense as grave as and equivalent to the felony of qualified theft. violation of Section 68 of PD 705, as amended by E.O. No. 277 while Potencio was discharged because
he was used as a state witness.
Articles 309 and 310 read: Art. 309. Penalties. - Any person guilty of theft shall be punished by:
Aggrieved, petitioner elevated the case to CA where he challenged the discharge of Potencio as
1. The penalty of prision mayor in its minimum and medium periods, if the value of the thing stolen is
a state witness on the ground that there is no absolute necessity for his testimony. Monge contested that
more 12,000 pesos but does not exceed 22,000 pesos; but if the value of the thing stolen exceeds the latter
it was Potencio who owned the lumbers and not him, that he was only hired by Potencio to transport the
amount, the penalty shall be the maximum period of the one prescribed in this paragraph, and one year for
lumbers to a sawmill. The appellate court dismissed his petition, hence, he filed a review on certiorari.
each additional ten thousand pesos, but the total of the penalty which may be imposed shall not exceed
twenty years. In such cases, and in connection with the accessory penalties which may be imposed and for Issue:
the purpose of the other provisions of this Code, the penalty shall be termed prision mayor or reclusion
temporal, as the case may be. (emphasis supplied) Whether or not Monge was guilty of the offense charged?

Held:
2. xxx Art. 310. Qualified theft. - The crime of theft shall be punished by the penalties next higher by two
degrees than those respectively specified in the next preceding articles xxx (emphasis supplied). Yes.

The contention of Monge is unavailing.


The actual market value of the 113 pieces of seized lumber was P67,630.9 Following Article 310 in relation
to Article 309, the imposable penalty should be reclusion temporal in its medium and maximum periods Under Section 68 of PD 705, as amended by E.O. No. 277, criminalizes two distinct and
or a period ranging from 14 years, eight months and one day to 20 years plus an additional period of four separate offences namely;
years for the excess of P47,630.
a. Cutting, gathering, collecting and removing of timber from alienable or disposable public
The minimum term of the indeterminate sentence10 imposable on Taopa shall be the penalty next lower land, or timber from alienable or disposable public land, or from private land without any authority; and
to that prescribed in the RPC. In this case, the minimum term shall be anywhere between 10 years and one b. The possession of timber or other forest products without legal documents required under the
day to 14 years and eight months or prision mayor in its maximum period to reclusion temporal in its existing laws and regulations.
minimum period.
In the first offense, the legality of the acts of cutting, gathering, collecting or removing timber
The maximum term shall be the sum of the additional four years and the medium period11 of reclusion or other forest products may be proven by the authorization duly issued by the DENR. The second
temporal in its medium and maximum periods or 16 years, five months and 11 days to 18 years, two offense, however, it is immaterial whether or not the cutting, gathering, collecting and removal of forest
months and 21 days of reclusion temporal. The maximum term therefore may be anywhere between 16 products are legal precisely because mere possession of forest products without the requisite documents
consummates the crime.

24
Petitioner cannot take refuge in his denial of ownership over the pieces of lumber found in his WHEREFORE, the decision of the Court of Appeals is AFFIRMED with the MODIFICATION as to the
possession nor his claim that he was merely hired by Potencio to provide the latter with assistance in penalty.
transporting the said lumber. PD 705 is a penal statute that punishes acts essentially malum prohibitum.
In other words, mere possession of timber or other forest products without the proper legal documents,
even absent malice or criminal intent is illegal.
23. ALVAREZ VS PICOP
22. PALLADA vs. PEOPLE G.R. No. 131270. March 17, 2000 G.R. No. 162243 November 29, 2006
CHICO-NAZARIO, J.:
FACTS:

DENR officers, assisted by the PNP, raided the warehouse of the Valencia Golden Harvest Corporation. FACTS:
The company is engaged in rice milling and trading. They found a large stockpile of lumber of varying PICOP was granted Timber License Agreement (TLA) No. 43 covering an area of 75,545 hectares in
sizes cut by a chain saw. As proof that the company had acquired the lumber by purchase, petitioner Surigao del Sur, Agusan del Sur, Compostela Valley, and Davao Oriental, to "terminate on April 25, 2002."
produced two receipts issued by R.L. Rivero Lumberyard of Maramag, Bukidnon, dated March 6 and 17, On 23 December 1999, DENR promulgated DENR Administrative Order (DAO) No. 99-53 mandating
1992. The DENR officers did not, however, give credit to the receipts considering that R. L. Rivero the conversion of the TLA into IFMA that is primarily aimed at sustaining the raw materials for the
Lumberyard's permit to operate had long been suspended. What is more, the pieces of lumber were cut continuous operation of the integrated wood processing plant of the company. Pursuant to this, PICOP
by chain saw and thus could not have come from a licensed sawmill operator. Accordingly, all the signified its intention to convert its TLA No. 43 into an Integrated Forest Management Agreement (IFMA).
lumber in the warehouse had been seized and the petitioner was charged with violation of §68 of P.D.No.
705, as amended. A Performance Evaluation Team was created to conduct performance evaluation indicating
violations by PICOP of such as the non-submission of its five-year forest protection plan and seven-year
RTC convicted the petitioner. The trial court did not give credence to the Certificates of Timber Origin reforestation plan as required by the DENR rules and regulations. Also there are alleged unpaid and
presented by petitioner since the lumber held by the company should be covered by Certificates of overdue forest charges of PICOP. It was the position of the DENR members that PICOP’s application for
Lumber Origin. His conviction was affirmed by the Court of Appeals. Hence, petitioner then filed a the IFMA conversion should undergo the process as provided in DAO No. 99-53. PICOP representative
petition for review before the Supreme Court. Atty. Caingat, however, claimed that "the conversion of TLA No. 43 into IFMA has already been
ISSUES: completed" and indicated that they had "no choice except to decline participation in the ongoing meeting
and bring our issues to the proper public and legal forum."
1. Whether separate certificates of origin should be issued for lumber and timber. 2. Whether the
presence of erasures in the certificate of timber origin render them valueless as evidence. Accordingly, the Secretary of DENR claims that further processing of PICOP’s application for
the conversion of TLA No. 43 cannot proceed until PICOP complies with the requirements. Insisting that
HELD:
the conversion of its TLA No. 43 had been completed, PICOP filed a Petition for Mandamus against then
Different certificates of origin are required for timber, lumber and non-timber forest products. The DENR Secretary Alvarez before the RTC of Quezon City. The RTC rendered a Decision granting PICOP’s
issuance of a separate certificate of origin for lumber is required in order to "pinpoint accountability and Petition for Mandamus. On appeal, CA affirmed the Decision of the RTC. Hence, this petition by the
responsibility for shipment of lumber . . . and to have uniformity in documenting the origin thereof." DENR assailing that the mandamus filed by PICOP should be out rightly dismissed on the ground that
RTC has no jurisdiction over the subject matter of the case since the acts questioned herein are related to
Even assuming that a Certificate of Timber Origin could serve as a substitute for Certificate of Lumber the licensing regulation and management of forest resources. Furthermore, RTC is prohibited to issue
Origin, the trial court and the Court of Appeals were justified in convicting petitioner, considering the TRO, writs of preliminary injunction and preliminary mandatory injunction as provided by PD605 and
numerous irregularities and defects found in the documents presented by the latter. The irregularities and RA8975.
discrepancies make the documents in which they are found not only questionable but invalid and, thus,
justified the trial court in giving no credence to the same. The presence of such glaring irregularities ISSUE: WON THE TRIAL COURT HAD JURISDICTION TO TAKE COGNIZANCE OF THIS CASE
negates the presumption that the CTOs were regularly executed by the DENR officials concerned. BECAUSE THE SUBJECT MATTER THEREOF PERTAINS TO THE EXCLUSIVE
ADMINISTRATIVE DOMAIN OF THE DENR SECRETARY
25
24. SOLEDAD DY v. CA, GR No. 121587, 1999-03-09
HELD: YES. The Petition filed before the trial court was one for mandamus with prayer for the issuance
of a writ of preliminary prohibitory and mandatory injunction with damages. Specifically, it sought to Facts: On May 31, 1993, the Mayor of Butuan City issued Executive Order No. 93-01 creating Task
compel the DENR Secretary to: (1) sign, execute and deliver the IFMA documents to PICOP; (2) issue the Force Kalikasan to combat "illegal logging, log smuggling or possession of and/or transport of illegally
corresponding IFMA number assignment; and (3) approve the harvesting of timber by PICOP from the cut or produced logs, lumber, flitches and other forest products" in that... city.
area of TLA No. 43. In its petition for mandamus, [PICOP] asserted that "DENR Secretary Alvarez acted
with grave abuse of discretion or in excess of his jurisdiction in refusing to perform his ministerial duty to On July 1, 1993, the members of the task force received confidential information that two truckloads of
sign, execute and deliver the IFMA contract and to issue the corresponding IFMA number to it." What is illegally cut lumber would be brought to Butuan City from the Ampayon-Taguibe-Tiniwisan area.
at stake is not the scope of the DENR jurisdiction but the manner by which it exercises or refuses to
exercise that jurisdiction. Forester Resurreccion Maxilom of the DENR issued a temporary seizure order and a seizure receipt for
the two vehicles and their cargo consisting of several pieces of lumber of different sizes and dimensions,
The courts have the duty and power to strike down any official act or omission tainted with grave but Lucero, the caretaker of the compound where they were seized,... refused to accept them. The seized
abuse of discretion. The 1987 Constitution is explicit in providing that judicial power includes not only lumber and vehicles were then taken to the City motorpool and placed in the custody of respondent
the duty of the courts of justice to settle actual controversies involving rights which are legally demandable Lausa.
and enforceable, but also to determine whether or not there has been grave abuse of discretion amounting
to lack or in excess of jurisdiction on the part of any branch or instrumentality of the government. Since For lack of claimants, DENR Regional Technical Director Raoul Geollegue recommended to the
PICOP alleges grave abuse of discretion on the part of the DENR Secretary, it behooves the court to Secretary on July 29, 1993 the forfeiture of the lumber and the two vehicles.
determine the same. An outright dismissal of the case would have prevented such determination.
On October 20, 1993, more than two months after the lumber had been forfeited, petitioner, claiming to
it is argued that PICOP’s immediate resort to the trial court was precipitate based on the doctrine be the owner of the lumber, filed a suit for replevin in the Regional Trial Court of Butuan City (Branch
of exhaustion of administrative remedies. This holds no water. The doctrine of exhaustion of administrative 5) for its recovery.
remedies is disregarded when there are circumstances indicating the urgency of judicial intervention,
which are averred to be extant in this case, citing PICOP’s employment of a sizable number of workers Before the court could act on his motion, he moved to dismiss and/or quash the writ of replevin on the
and its payment of millions in taxes to the government. Moreover, contrary to [the DENR Secretary’s] ground that the lumber in question, having been seized and forfeited by the DENR... pursuant to P.D.
claim, the approval of an application for IFMA conversion is not purely discretionary on the part of the No. 705, as amended (Revised Forestry Code), was under its custody and, therefore, resort should first
DENR Secretary since the approval of an IFMA conversion depends upon compliance with the be made to the DENR.
requirements provided under DAO No. 99-53. Even assuming, arguendo, that the approval of an IFMA
conversion involves the exercise of discretion by the DENR Secretary, the writ of mandamus may be
Issues:
issued to compel the proper exercise of that discretion where it is shown that there was grave abuse of
discretion, manifest injustice, or palpable excess of authority.
WITH DUE RESPECT RESPONDENT COURT OF APPEALS ERRED IN RULING THAT THE
VERIFICATION MADE BY LORENCIO DY AND NOT BY PETITIONER SOLEDAD Y. DY WAS
DENR Sec is wrong in invoking the provisions of PD605 and RA8975. These statutes merely
INSUFFICIENT TO JUSTIFY THE ISSUANCE OF THE REPLEVIN WRIT.
proscribe the issuance of temporary restraining orders and writs of preliminary injunction and preliminary
mandatory injunction. They cannot, under pain of violating the Constitution, deprive the courts of authority
THE RESPONDENT COURT OF APPEALS ERRED IN RULING THAT A COUNTERBOND IN
to take cognizance of the issues raised in the principal action, as long as such action and the relief sought
REPLEVIN WHICH IS EFFECTIVE FOR ONLY ONE YEAR IS VALID TO CAUSE THE RETURN
are within their jurisdiction.
OF THE PROPERTY TO DEFENDANT.
Hence, the outright dismissal of the mandamus prayed for by DENR herein is hereby denied.
THE RESPONDENT COURT OF APPEALS ERRED IN GIVING DUE COURSE TO PRIVATE
RESPONDENT'S PETITION FOR CERTIORARI.
26
The appeal is without merit. The threshold question is whether the Regional Trial Court could in fact 25. TIGOY vs. CA Case Digest
take cognizance of the replevin suit, considering that the object was the recovery of lumber seized and RODOLFO TIGOY vs. COURT OF APPEALSG.R. No. 144640. June 26, 2006
forfeited by law enforcement agents of the DENR pursuant to P.D. No. 705 FACTS:
Nestor Ong, who had been engaged in the trucking business in Iligan City since 1986, was allegedly
(Revised Forestry Code), as amended by Executive Order No. 277. introduced by his friend Gamad Muntod to Lolong Bertodazo who signified his intent to rent the trucks
of Ong to transport construction materials from Larapan, Lanao del Norte to Dipolog City. A Contract to
Ruling: Transport was supposedly entered into between Ong and Bertodazo.In the evening of October 3, 1993,
Ong allegedly ordered Nestor Sumagang and petitioner Rodolfo Tigoy to bring the two trucks to Lolong
The rule is that a party must exhaust all administrative remedies before he can resort to the courts. Bertodazo in Larapan, Lanao del Norte. He instructed the two drivers to leave the trucks in Larapan for
the loading of the construction materials by Lolong Bertodazo. Thus, after meeting with Bertodazo,
Hence, if a remedy within the administrative machinery can still be resorted to by giving the Sumagang and petitioner Tigoy allegedly went home to return to Larapan at four o'clock in themorning
administrative officer concerned every opportunity to decide on a matter that comes within his the next day. When they arrived, the trucks had been laden with bags of cement and were half-covered
jurisdiction then such... remedy should be exhausted first before a court's judicial power can be sought. with canvas. That same morning of October 4, 1993, the Ozamis City police received a report that two
trucks, a blue and green loaded with cement, did not stop at the checkpoint. Thus, some police officers
As petitioner clearly failed to exhaust available administrative remedies, the Court of Appeals correctly boarded their patrol vehicle to intercept the two trucks. Upon inspection, the police officers discovered
set aside the assailed orders of the trial court granting petitioner's application for a replevin writ and piles of sawn lumber beneath the cement bags in both trucks. The police officers inquired if the drivers
denying private respondent's motion to dismiss. had a permit for the lumber but the latter could not produce any. After an investigation was held by the
police and the DENR office in the city, an Information was filed against Nestor Ong, Sumagang, Lolong
Having been... forfeited pursuant to P.D. No. 705, as amended, the lumber properly came under the Bertodazo and petitioner Tigoy for possession of forest products without legal permit in violation of
custody of the DENR and all actions seeking to recover possession thereof should be directed to that Section 68 of Presidential Decree 705, as amended by Executive Order No. 277, Series of 1987, in
agency. relation to Article 309 and 310 of the Revised Penal Code. Ong and petitioner Tigoy entered pleas of not
guilty during the arraignment. After trial, the Regional Trial Court found both Ong and Tigoy guilty. On
appeal, Ong was acquitted while Tigoy’s conviction was upheld.
WHEREFORE, the decision of the Court of Appeals, dated January 19, 1995, and its Resolution, dated
July 26, 1995, in CA-G.R. SP 33099 are AFFIRMED with the modification that the complaint for
ISSUE:
recovery of personal property is DISMISSED.
Is Tigoy guilty of possession of forest products without permit?
Principles:
HELD:
Yes. There are two ways of violating the said Section 68: 1) by cutting, gathering and/or collecting
Section 8 of P.D. No. 705, as amended, provides:
timber or other forest products without a license; and, 2) by possessing timber or other forest products
without the required legal documents. Petitioner was charged with and convicted of transporting lumber
SEC. 8. Review. ¾ All actions and decisions of the Director are subject to review, motu propio or upon
without a permit which is punishable under Section 68 of the Code. The appellant, Sumagang and the
appeal of any person aggrieved thereby, by the Department Head whose decision shall be final and
rest of their companions were apprehended by the police officers in flagrante delicto as they were
executory after the lapse of thirty (30) days from... receipt by the aggrieved party of said decision, unless
transporting the subject lumber from Larapan to Dipolog City. Tigoy contends that he did not know that
appealed to the President in accordance with Executive Order No. 19, series of 1966. The Decision of the
the truck was loaded with timber without the necessary permit. However, the circumstances shows
Department Head may not be reviewed by the courts except through a special civil action for certiorari
otherwise. Why would the drivers refuse to stop when required? Did they fear inspection of their cargo?
or... prohibition.
Why would "S.O.P." (which in street parlance is grease money) be offered to facilitate the passage of the
trucks? The only logical answer to all these questions is that the drivers knew that they were carrying
contraband lumber. In offenses considered as mala prohibita or when the doing of an act is prohibited by

27
a special law such as in the present case, the commission of the prohibited act is the crime itself. It is really intimidated by Santiago. If at all, this could only make petitioner administratively liable for his
sufficient that the offender has the intent to perpetrate the act prohibited by the special law, and that it is acts. It is not enough to convict him under Section 68 of PD 705.
done knowingly and consciously. Direct proof of previous agreement to commit an offense is not
necessary to prove conspiracy. Conspiracy may be proven by circumstantial evidence. It may be deduced
from the mode, method and manner by which the offense is perpetrated, or inferred from the acts of the
accused when such acts point to a joint purpose and design, concerted action and community of interest.
It is not even required that the participants have an agreement for an appreciable period to commence it. 27. [G.R. No. L-46772. February 13, 1992.]

THE PEOPLE OF THE PHILIPPINES, petitioner, vs. COURT OF FIRST INSTANCE OF


26. AQUINO v. PEOPLE GR No. 165448 July 27, 2009 QUEZON (BRANCH VII), GODOFREDO ARROZAL AND LUIS FLORES, respondents.

FACTS:

The Teacherâs Camp filed with the DENR an application to cut down 14 dead Benguet pine trees within MEDIALDEA, J p:
their area in Baguio City. The trees were to be used for the repairs of Teachers Camp. After the
Doctrine:
inspection of the trees to be cut, the Executive Director of the DENR issued a permit allowing the cutting
of 14 trees. Thereafter, a group of forest rangers received information that pine trees were being cut When an accused invokes in a motion to quash the ground that the facts charged do not constitute an
without proper authority at the Teacherâs Camp. They went to the site where they found petitioner offense (Rule 117, Sec. 2[a] Rules of Court), the sufficiency of the Information hinges on the question of
Aquino, a forest ranger from CENRO, another forest ranger, two supervisors, and two sawyers. The whether the facts alleged, if hypothetically admitted, meet the essential elements of the offense defined in
forest rangers found 23 tree stumps, out of which only 12 were covered by the permit. An information the law.
was then filed against the five individuals for cutting without permit the nine (9) pine trees in conspiracy.
The trial court ruled that despite the existence of a permit, the trees cut exceeded the allowed number of ***The elements of the crime of qualified theft of logs are: 1) That the accused cut, gathered, collected or
the trees authorized to be cut and that the cutting of trees went beyond the period stated in the permit. removed timber or other forest products; 2) that the timber of other forest products cut, gathered, collected
Nonetheless, all of the accused have been acquitted in the trial court and on appeal, except for the or removed belongs to the government or to any private individual; and 3) that the cutting, gathering,
petitioner. Petitionerâs defense was that he was merely sent to supervise the cutting of trees at the collecting or removing was without authority under a license agreement, lease, license, or permit granted
by the state.
Teacherâs Camp and he was not aware of the trees covered by the permit. However, he still supervised
the cutting of trees without procuring a copy of the vicinity map used in the inspection of the trees to be Facts:
cut. He claimed that he could not prevent the overcutting of trees because he was just alone and that he
feared one of the sawyers, Santiago. This petition seeks the annulment of the order of the CFI of Quezon dismissing the information
filed therein.
ISSUE: WoN petitioner is guilty beyond reasonable doubt of violation of Section 68 of PD 705
The private respondents were charged with the crime of qualified theft of logs, defined and
HELD: NO. Section 68 of PD 705 punishes anyone who shall cut, gather, collect or remove timber or punished under Section 68 of Presidential Decree No. 705, otherwise known as the Revised Forestry Code
other forest products from any forest land, or timber from alienable or disposable public land, or from of the Philippines, in an information which read:
private land, without any authority. In this case, petitioner was charged by CENRO to supervise the
implementation of the permit. He was not the one who cut, gathered, collected or removed the pine trees On March 23, 1977, the named accused filed a motion to quash the information on two (2)
within the contemplation of Section 68 of PD 705. He was not in possession of the cut trees because the grounds, to wit: (1) that the facts charged do not constitute an offense; and, (2) that the information does
lumber was used by Teachers Camp for repairs. Petitioner could not likewise be convicted of conspiracy not conform substantially to the prescribed form. The Trial court dismissed the information on the grounds
to commit the offense because all his co-accused were acquitted of the charges against them. Petitioner invoked and the reconsideration sought was denied.
may have been remiss in his duties when he failed to restrain the sawyers from cutting trees more than
Hence this petition.
what was covered by the permit. As the CA ruled, petitioner could have informed his superiors if he was

28
Issue: WoN the information charged an offense. PHILIPPINE MINING ACT CASES

Held:
28. Atok Big Wedge Mining Co. v. Intermediate Appellate Court
YES. The Court agree with the petitioner that the information substantially alleged all the Facts:
elements of the crime of qualified theft of logs as described in Section 68 of P.D. 705. While it was
admitted that the information did not precisely allege that the taking of the logs in question was "without Private respondents are in an open, notorious and exclusive possession of the land; Petitioner Atok
the consent of the state," nevertheless, said information expressly stated that the accused "illegally cut, alleges otherwise due to supposed mining claim
gather, take, steal and carry away therefrom, without the consent of said owner and without any authority
under a license agreement, lease, license or permit, sixty (60) logs of different species. . . ." Since only the
Held:
state can grant the lease, license, license agreement or permit for utilization of forest resources, including
timber, then the allegation in the information that the asportation of the logs was "without any authority" Mere recording of a mining claim, without performing annual work obligation, does not convert the land
under a license agreement, lease, license or permit, is tantamount to alleging that the taking of the logs was into mineral land. The recording only operates as reservation to the registrant exclusive rights to
without the consent of the state. undertake mining activities. Thus, if no minerals are extracted therefrom, the land is not mineral land and
registration is not precluded by such recorded claim.
When an accused invokes in a motion to quash the ground that the facts charged do not constitute
an offense (Rule 117, Sec. 2[a] Rules of Court), the sufficiency of the Information hinges on the question
The respondent appellate court, on its part, correctly considered inadequate, however, the mere recording
of whether the facts alleged, if hypothetically admitted, meet the essential elements of the offense defined
of petitioners mining claims in the Mining Recorder of Benguet and the corresponding, albeit religious,
in the law.
payment of annual assessment fees therefor, to vest in petitioner ownership rights over the land in
The failure of the information to allege that the logs taken were owned by the state is not fatal. question. Truly, under Executive Order No. 141, the payment of annual assessment fees is only proof of
The fact that only the state can grant a license agreement, license or lease does not make the state the compliance with the charges imposed by law and does not constitute proof of actual assessment work on
owner of all the logs and timber products produced in the Philippines including those produced in private the mining land concerned.
woodlands. While it is only the state which can grant a license or authority to cut, gather, collect or remove
forest products it does not follow that all forest products belong to the state. In the just cited case, private Under the Philippine Bill of 1902, the mining claim holder, upon locating and recording of his claim, has
ownership of forest products grown in private lands is retained under the principle in civil law that the right to acquire for himself all mineral deposits found within his claim to the exclusion of everyone,
ownership of the land includes everything found on its surface. including the Government. Such rights are necessarily possessory as they are essentially utilitarian and
exploitative. Such rights accruing to the mining claim locator are personal to him in the sense that no
Ownership is not an essential element of the offense as defined in Section 60 of P.D. No. 705. conclusion as to the nature of the land may definitively be made based solely on the fact that a mining
Thus, the failure of the information to allege the true owner of the forest products is not material, it was claim has been recorded as regards a particular land. However, insofar as his rights are exclusive and no
sufficient that it alleged that the taking was without any authority or license from the government. other person may undertake mining activities on a recorded mining claim, unless the same has been
Dispositive Portion: abandoned or the works thereon not done, the mining locators rights are also protected against adverse
mining claims of third persons. He also has the right to immediately or eventually secure a patent on his
ACCORDINGLY, the petition is GRANTED. The questioned order of the trial court dismissing the mining claim and in the event that he postpones securing a patent, his rights to exclusive possession and
information is SET ASIDE. Criminal Case No. 1591 is reinstated. exploitation of his mining claim subsist for as long as he complies with the continuing requirement of
annually performing work or undertaking improvements at the mine site. Insofar as the Philippine Bill of
1902 does not provide a specific time within which the mining claim holder must secure a patent, his
rights to possession and use of the mining land appear to be unconditional, the option not at all to secure
a patent being available to him in the absence of a deadline or ultimatum therefor. The Philippine Bill of
1902, however, did not foreclose a subsequent act on the part of the State to limit the time within which
the said patent must be secured under threat of forfeiture of rights provided for under the Philippine Bill
29
of 1902. Thus, in the sense that the rights of a mining claim holder may in the future be curtailed by Lots 6-9 to his children by Mamaya Balbalio and Jaime Alberto, respectively, in 1964.
failure to obtain a patent, especially if we recall that Section 36 of the said Bill itself foretold the The application was separately opposed by Benguet Consolidated, Inc. as to Lots 1-5, Atok Big Wedge
subsequent promulgation of regulations regarding mining claims, such rights cannot also be said to be Corporation, as to Portions of Lots 1-5 and all of Lots 6-9, and by the Republic of the Philippines,
truly unconditional or absolute. through the Bureau of Forestry Development, as to lots 1-9.

In support of the application, both Balbalio and Alberto testified that they had acquired the subject land
The process of recording mining claims could not have been intended to be the operative act of
by virtue of prescription; Balbalio claimed to have received Lots 1-5 from her father shortly after the
classifying lands into mineral lands. The recording of a mining claim only operates to reserve to the
Liberation.
registrant exclusive rights to undertake mining activities upon the land subject of the claim. The power to
classify lands into mineral lands could not have been intended under the Philippine Bill of 1902 to be
vested in just anyone who records a mining claim. In fact, this strengthens our holding that the rights of a Benguet opposed on the ground that the “June Bug” mineral claim covering Lots 1-5 was sold to it on
mining claimant are confined to possessing the land for purposes of extracting therefrom minerals in September 22, 1934, by the successors-in-interest of James Kelly, who located the claim in September
exclusion of any or all other persons whose claims are subsequent to the original mining locator. Thus, if 1909 and recorded it on October 14, 1909. From the date of its purchase, Benguet had been in actual,
no minerals are extracted therefrom, notwithstanding the recording of the claim, the land is not continuous and exclusive possession of the land in concept of owner, as evidenced by its construction of
mineral land and registration thereof is not precluded by such recorded claim. Thus, in the case at adits, its affidavits of annual assessment, its geological mappings, geological samplings and trench side
bench, the mining claimant, who had failed to comply with the annual minimum labor requirement, cuts, and its payment of taxes on the land.
could not, all the more, be expected to have extracted minerals from the mining location. Utter lack of
proof of even its potential deposits on the part of the petitioner, thus, does not surprise us at all.
For its part, Atok alleged that a portion of Lots 1-5 and all of Lots 6-9 were covered by the Emma and
Thus, it can be said (1) that the rights under the Philippine Bill of 1902 of a mining claim holder over his Fredia mineral claims located by Harrison and Reynolds on December 25, 1930, and recorded on
claim has been made subject by the said Bill itself to the strict requirement that he actually performs January 2, 1931, in the office of the mining recorder of Baguio. These claims were purchased from these
work or undertakes improvements on the mine every year and does not merely file his affidavit of annual locators on November 2, 1931, by Atok, which has since then been in open, continuous and exclusive
assessment, which requirement was correctly identified and declared in E.O. No. 141; and (2) that the possession of the said lots as evidenced by its annual assessment work on the claims, such as the boring
same rights have been terminated by P.D. No. 1214, a police power enactment, under which non- of tunnels, and its payment of annual taxes thereon.
application for mining lease amounts to waiver of all rights under the Philippine Bill of 1902 and
application for mining lease amounts to waiver of the right under said Bill to apply for patent. In the light The Bureau of Forestry Development also interposed its objection, arguing that the land sought to be
of these substantial conditions upon the rights of a mining claim holder under the Philippine Bill of 1902, registered was covered by the Central Cordillera Forest Reserve under Proclamation No. 217 dated
there should remain no doubt now that such rights were not, in the first place, absolute or in the nature of February 16, 1929. Moreover, by reason of its nature, it was not subject to alienation under the
ownership, and neither were they intended to be so. Constitutions of 1935 and 1973.

29. Republic vs. CA and De La Rosa


The trial court denied the application, holding that the applicants had failed to prove their claim of
Republic of the Philippines, Benguet & Atok vs. Court of Appeals & De La Rosa possession and ownership of the land sought to be registered.
G.R. No. L-43938, April 15, 1988

Cruz, J.: The applicants appealed to the respondent court, which reversed the trial court and recognized the claims
of the applicant, but subject to the rights of Benguet and Atok respecting their mining claims. In other
FACTS: These consolidated cases arose from the application for registration of a parcel of land filed on
words, the Court of Appeals affirmed the surface rights of the de la Rosas over the land while at the
February 11, 1965, by Jose de la Rosa on his own behalf and on behalf of his three children, Victoria,
same time reserving the sub-surface rights of Benguet and Atok by virtue of their mining claims. Both
Benjamin and Eduardo. The land, situated in Tuding, Itogon, Benguet Province, was divided into 9 lots
Benguet and Atok have appealed to this Court, invoking their superior right of ownership.
and covered by plan Psu-225009. According to the application, Lots 1-5 were sold to Jose de la Rosa and

30
ISSUE: WHETHER OR NOT APPLICANTS ALL SURNAMED DELA ROSA HAVE SUPERIOR The flaw in the reasoning of the respondent court is in supposing that the rights over the land could be
RIGHTS OF OWNERSHIP OVER THE SURFACE RIGHTS OVER THE LAND IN QUESTION used for both mining and non-mining purposes simultaneously. The correct interpretation is that once
WHILE OPPOSITORS BENGUET CONSOLIDATED, INC. AND ATOK BIG WEDGE MINING minerals are discovered in the land, whatever the use to which it is being devoted at the time, such
COMPANY ARE RESERVED OF THEIR SUB-SURFACE RIGHTS BY VIRTUE OF THEIR use may be discontinued by the State to enable it to extract the minerals therein in the exercise of
MINING CLAIM AS DECIDED BY THE RESPONDENT COURT. its sovereign prerogative. The land is thus converted to mineral land and may not be used by any
private party, including the registered owner thereof, for any other purpose that will impede the mining
operations to be undertaken therein. The Regalian doctrine then extends not only to land but also to “all
HELD: NO. Our holding is that Benguet and Atok have exclusive rights to the property in question by natural wealth that may be found in the bowels of the earth.”
virtue of their respective mining claims which they validly acquired before the Constitution of 1935
prohibited the alienation of all lands of the public domain except agricultural lands, subject to vested 1. MARCOS B. COMILANG vs. HON. GENEROSO A. BUENDIA
rights existing at the time of its adoption. The land was not and could not have been transferred to the
private respondents by virtue of acquisitive prescription, nor could its use be shared simultaneously by G.R. No. L-24757 October 25, 1967
them and the mining companies for agricultural and mineral purposes. It is true that the subject property
was considered forest land and included in the Central Cordillera Forest Reserve, but this did not impair Facts:
the rights already vested in Benguet and Atok at that time. Such rights were not affected either by the
stricture in the Commonwealth Constitution against the alienation of all lands of the public domain  Nicolas Comilang staked a mining claim known as the "Bua Fraction Mineral Claim" over a
except those agricultural in nature for this was made subject to existing rights. The perfection of the parcel of land in Tuding, Benguet, Mountain Province, with an area of 76,809 square meters,
mining claim converted the property to mineral land and under the laws then in force removed it from more or less. He stopped the exploration but continue to live in the house built on a portion of the
the public domain. By such act, the locators acquired exclusive rights over the land, against even the land with his wife and other relatives. In 1918, Macario Comilang and his relatives also settled
government, without need of any further act such as the purchase of the land or the obtention of a patent on a portion of the land with an area of about one (1) hectare, for residential and agricultural
purposes.
over it. As the land had become the private property of the locators, they had the right to transfer the
 Surface rights over the area embraced in the original Bua Fraction Mineral Claim of Nicolas
same, as they did, to Benguet and Atok. The Court of Appeals justified this by saying there is “no Comilang soon became the subject of litigation in an action to quiet title filed in RTC of Baguio
conflict of interest” between the owners of the surface rights and the owners of the sub-surface rights. by the other heirs against Macario claiming that they bought the rights and interest of Nicolas
This is rather doctrine, for it is a well-known principle that the owner of piece of land has rights not Comilang in the old mining claim.
only to its surface but also to everything underneath and the airspace above it up to a reasonable  The court dismissed both claims of ownership of petitioner and respondent declaring the area as
height. Under the aforesaid ruling, the land is classified as mineral underneath and agricultural on the a public land, but recognized the possession of Macario Comilang over 1½ hectares which was
surface, subject to separate claims of title. This is also difficult to understand, especially in its practical declared for taxation purposes but later on levied and sold at a public auction to satisfy a money
application. judgment obtained by spouses Jose Coloma and Eugenia Rumbaoa against Macario filed in the
RTC of Baguio. The spouses were the purchasers in the auction and the certificate of sale was
issued in favor of them.
 In the meantime, an application for lode patent covering the Bua Fraction Mineral Claim was
The Regalian doctrine which, as its name implies, is intended for the benefit of the State, not of private filed with the Bureau of Mines. Abdon Delenela and his co-heirs filed their opposition to the
persons. The rule simply reserves to the State all minerals that may be found in public and even private application. Pending application, Delenela filed an action for the determination of their rights on
land devoted to "agricultural, industrial, commercial, residential or (for) any purpose other than mining." the land in the RTC of Baguio to which the said court awarded one-half in undivided share in the
mineral claim in favor of Marcos and the other half also in undivided share in favor of Abdon
Thus, if a person is the owner of agricultural land in which minerals are discovered, his ownership of
Delenela and co-heirs. Delenela, with the knowledge and conformity of Marcos Comilang,
such land does not give him the right to extract or utilize the said minerals without the permission of the redeemed and bought from the Coloma spouses, the latter's rights, title, interest and claim to the
State to which such minerals belong. 1-1/2 hectares of land acquired under the certificate of sale. A writ of possession was issued in
their favor.
 In a petition for certiorari with PI filed in the RTC of Baguio, the wife of Marcos questioned the
power of municipal court to issue said writ of possession on two grounds (1) that conjugal
31
property had been levied upon and sold in the execution sale, and her share therein is affected; 1980s, Diwalwal has been stormed by conflicts brought about by numerous mining claims over it. On
and (2) that there can be no severance of surface rights over a mineral claim located under the March 10, 1986, Marcopper Mining Corporation (MMC) was granted an Exploration Permit (EP 133) by
Philippine Bill of 1902, and petitioner argued that the sheriff could not have validly sold the the Bureau of Mines and Geo-Sciences (BMG). A long battle ensued between Apex and MMC with the
surface rights in the execution sale of June 1, 1957. The court rendered a decision in said case, latter seeking the cancellation of the mining claims of Apex on the ground that such mining claims were
holding that the writ of possession issued by the respondent Municipal Judge was within his within a forest reservation (Agusan-Davao-Surigao Forest Reserve) and thus the acquisition on mining
competence and jurisdiction. On appeal, the decision became final. rights should have been through an application for a permit to prospect with the BFD and not through
 For a second time, a petition for certiorari and mandamus with PI was instituted by Marcos registration of a DOL with the BMG. When it reached the SC in 1991, the Court ruled against Apex holding
Comilang in the RTC of Baguio City seeking the annulment of the order granting the alias writ
that the area is a forest reserve and thus it should have applied for a permit to prospect with the BFD. On
of possession in favor of Delenela and Perez, and again the Court of First Instance of Baguio
threw out the petition in its order dated October 22, 1964. Hence the petition. February 16 1994, MMC assigned all its rights to EP 133 to Southeast Mindanao Gold Mining Corporation
(SEM), a domestic corporation which is alleged to be a 100%-owned subsidiary of MMC. Subsequently,
BMG registered SEM’s Mineral Production Sharing Agreement (MPSA) application and the Deed of
Issue: whether or not the final certificate of sale conveying the land described in Tax Declaration No. 4771
to the purchasers in the execution sale is not a valid disposition of a portion of the public domain, and Assignment. Several oppositions were filed. The Panel of Arbitrators created by the DENR upheld the
specially in view of the subsequent issuance of a mineral lode patent over the Bua Mineral Claim by the validity of EP 133. During the pendency of the case, DENR AO No. 2002-18 was issued declaring an
Director of Mines (Patent issued on November 7, 1966) whereby full ownership not only of the minerals emergency situation in the Diwalwal Gold Rush Area and ordering the stoppage of all mining operations
therein but also of the surface ground have been conveyed to the patentee thereof, and, therefore, the therein.
Municipal Court of Baguio City may no longer eject them from the land.

Held: No. The 1-½ hectares portions of the Bua Fraction Mineral Claim described in Tax Declaration No. Issues:
4771 in the name of herein appellant was levied upon and sold at public auction to satisfy the money
judgment against him in Civil Case No. 1433 of the Municipal Court of Baguio City, and the corresponding 1. W/N EP 133 and its subsequent transfer to SEM is valid.
certificate of sale was issued in favor of the judgment creditors. Interest acquired under like certificates of
sale alone has been described as more than a lien on the property, more than an equitable estate, an inchoate 2. W/N the DENR Secretary has authority to issue DAO 66 declaring 729 hectares of the areas covered by
legal title to the property. the Agusan-Davao-Surigao Forest Reserve as non-forest lands and open to smallscale mining purposes.

3. Who (among petitioners Apex and Balite) has priority right over Diwalwal?
The validity of that sale was questioned when the Municipal Court ordered the eviction of appellant from
the land sold on execution, and the Supreme Court declared in L-18897 that the sale was valid. The sale Held/Ratio:
operated to divest appellant of his rights to the land which vested in the purchasers at the auction sale. The
parties herein subsequently litigated their rights to the mineral claim in Civil Case No. 735 of the Court of 1. INVALID. One of the terms and conditions of EP 133 is: “That this permit shall be for the exclusive
First Instance of Baguio City, and on the basis of their amicable agreement (appellant was a party in the use and benefit of the permittee or his duly authorized agents and shall be used for mineral exploration
case), the court declared the Bua Mineral Claim co-ownership property of the parties thereto "except the purposes only and for no other purpose.” While it may be true that SEM is a 100% subsidiary corporation
improvements existing thereon". of MMC, there is no showing that the former is the duly authorized agent of the latter. As such, the
assignment is null and void as it directly contravenes the terms and conditions of the grant of EP 133. a.
There is no room for doubt, therefore, that the right to possess or own the surface ground is separate and The Deed of Assignment was a total abdication of MMC’s rights over the permit. It is not a mere grant of
distinct from the mineral rights over the same land. And when the application for lode patent to the mineral authority to SEM as agent. b. Reason for the stipulation. Exploration permits are strictly granted to entities
claim was prosecuted in the Bureau of Mines, the said application could not have legally included the or individuals possessing the resources and capability to undertake mining operations. Without such a
surface ground sold to another in the execution sale. Consequently, We have to declare that the patent condition, non-qualified entities or individuals could circumvent the strict requirements under the law by
procured thereunder, at least with respect to the 1-½ hectares sold in execution pertains only to the mineral the simple expediency of acquiring the permit from the original permittee. c. Separate personality. The
right and does not include the surface ground of the land in question.
fact that SEM is a 100% subsidiary of MMC does not automatically make it an agent of MMC. A
corporation is an artificial being invested by law with a personality separate and distinct from persons
2. APEX MINING CO., INC. V. SOUTHEAST MINDANAO GOLD MINING CORP. (2006) composing it as well as from that of any other legal entity to which it may be related. Absent any clear
Facts: The case involves the “Diwalwal Gold Rush Area” (Diwalwal), a rich tract of mineral land located proof to the contrary, SEM is a separate and distinct entity from MMC. d. Doctrine of piercing the
inside the Agusan-Davao-Surigao Forest Reserve in Davao del Norte and Davao Oriental. Since the early corporate veil inapplicable. Only in cases where the corporate fiction was used as a shield for fraud,

32
illegality or inequity may the veil be pierced and removed. The doctrine of piercing the corporate veil Secretary of the DENR and created the Provincial Mining Regulatory Board (PMRB) under the DENR
cannot therefore be used as a vehicle to commit prohibited acts. The assignment of the permit in favor of Secretary’s direct supervision and control. The statute also authorized the PMRB to declare and set aside
SEM is utilized to circumvent the condition of nontransferability of the exploration permit. To allow SEM small-scale mining areas subject to review by the DENR Secretary and award mining contracts to small-
to avail itself of this doctrine and to approve the validity of the assignment is tantamount to sanctioning an scale miners under certain conditions.
illegal act which is what the doctrine precisely seeks to forestall. e. PD 463 requires approval of Secretary DENR Secretary Fulgencio S. Factoran issued Department Administrative Order (DAO) No. 66, declaring
of DENR. Also, PD 463 (Mineral Resources Development Decree), which is the governing law when the 729 hectares of the Diwalwal area as non-forest land open to small-scale mining. The issuance was made
assignment was executed, explicitly requires that the transfer or assignment of mining rights, including the pursuant to the powers vested in the DENR Secretary by Proclamation No. 369, which established the
right to explore a mining area, must be with the prior approval of the Secretary of DENR. Such is not Agusan-Davao-Surigao Forest Reserve.
present in this case. f. EP 133 expired by non-renewal. Although EP 133 was extended for 12 months until
July 6, 1994, MMC never renewed its permit prior and after its expiration. With the expiration of EP 133 On April 1, 1997, Provincial Mining Regulatory Board of Davao passed Resolution No. 26, Series of 1997,
on July 6, 1994, MMC lost any right to the Diwalwal Gold Rush Area. SEM, on the other hand, has not authorizing the issuance of ore transport permits (OTPs) to small-scale miners operating in the Diwalwal
mines.
acquired any right to the said area because the transfer of EP 133 in its favor is invalid. Hence, both MMC
and SEM have not acquired any vested right over the area covered by EP 133. Thus, on May 30, 1997, petitioner filed a complaint for damages before the Regional Trial Court of Makati
City, Branch 61, against the DENR Secretary and PMRB-Davao. SEM alleged that the illegal issuance of
2. NO. The DENR Secretary has no power to convert forest reserves into non-forest reserves. Such power the OTPs allowed the extraction and hauling of P60,000.00 worth of gold ore per truckload from SEM's
is vested with the President. The DENR Secretary may only recommend to the President which forest mining claim.
reservations are to be withdrawn from the coverage thereof. Thus, DAO No. 66 is null and void for having
been issued in excess of the DENR Secretary’s authority. Petitioner then filed a special civil action for certiorari, prohibition and mandamus before the Court of
Appeals against PMRB-Davao, the DENR Secretary and Balite Communal Portal Mining Cooperative
3. (Since it’s been held that neither MMC nor SEM has any right over Diwalwal, it is thus necessary to (BCPMC), which represented all the OTP grantees. It prayed for the nullification of the above-quoted
make a determination of the existing right of the remaining claimants, petitioners Apex and Balite, in the Memorandum Order No. 97-03 on the ground that the “direct state utilization” espoused therein would
dispute.) The issue on who has priority right over Diwalwal is deemed overtaken by the issuance of effectively impair its vested rights under EP No. 133.
Proclamation 297 and DAO No. 2002-18, both being constitutionally-sanctioned acts of the Executive
The Court of Appeals dismissed the petition. It ruled that the DENR Secretary did not abuse his discretion
Branch. Mining operations in the Diwalwal Mineral Reservation are now, therefore, within the full control in issuing Memorandum Order No. 97-03 since the same was merely a directive to conduct studies on the
of the State through the executive branch. Pursuant to Sec. 5 of RA 7942, the State can either: (1) directly various options available to the government for solving the Diwalwal conflict.
undertake the exploration, development and utilization of the area or (2) opt to award mining operations
in the mineral reservation to private entities including petitioners Apex and Balite, if it wishes. The ISSUE: Whether or not the Court of Appeals erred when it concluded that the assailed memorandum order
exercise of this prerogative lies with the Executive Department over which courts will not interfere. did not adopt the “direct state utilization scheme” in resolving the Diwalwal dispute.

3. SOUTHEAST MINDANAO GOLD MINING CORPORATION v. BALITE PORTAL MINING RULING:


COOPERATIVE No. The challenged MO 97-03 did not conclusively adopt "direct state utilization" as a policy in resolving
G.R. No. 135190 April 3, 2002 the Diwalwal dispute. The terms of the memorandum clearly indicate that what was directed thereunder
was merely a study of this option and nothing else. Contrary to petitioner's contention, it did not grant any
FACTS: management/operating or profit-sharing agreement to small-scale miners or to any party, for that matter,
but simply instructed the DENR officials concerned to undertake studies to determine its feasibility.
The instant case involves a rich tract of mineral land situated in the Agusan-Davao-Surigao Forest Reserve
known as the “Diwalwal Gold Rush Area.” Located at Mt. Diwata in the municipalities of Monkayo and Further, there is no merit in petitioner's assertion that MO 97-03 sanctions violation of mining laws by
Cateel in Davao Del Norte, the land has been embroiled in controversy since the mid-80’s due to the allowing illegal miners to enter into mining agreements with the State, and it must likewise be pointed out
scramble over gold deposits found within its bowels. that under no circumstances may petitioner's rights under EP No. 133 be regarded as total and absolute.
On March 10, 1988, Marcopper Mining Corporation (Marcopper) was granted Exploration Permit No. 133 Petitioner's first assigned error is baseless and premised on tentative assumptions. Petitioner cannot claim
(EP No. 133) over 4,491 hectares of land, which included the hotly-contested Diwalwal area. any absolute right to the Diwalwal mines pending resolution of the Consolidated Mines cases, much less
ask us to assume, at this point, that respondent BCMC and the other mining firms are illegal miners. These
Not long thereafter, Congress enacted on June 27, 1991 Republic Act No. 7076, or the People’s Small- factual issues are to be properly threshed out in CA G.R. SP Nos. 61215 and 61216, which have yet to be
Scale Mining Act. The law established a People’s Small-Scale Mining Program to be implemented by the
33
decided by the Court of Appeals. Any objection raised against MO 97-03 is likewise premature at this 3. AO No.82 declares that failure to submit the LETTER OF INTENT and Mineral Production Sharing
point, inasmuch as it merely ordered a study of an option which the State is authorized by law to undertake. Agreement within 2 years from the date of effectivity of said guidelines shall cause an abandonment of
their mining, quarry, sand and gravel permits.
There is no need to rule on the matter of the OTPs, considering that the grounds invoked by petitioner for
invalidating the same are inextricably linked to the issues raised in the Consolidated Mines cases. On 13 November 1991, Continental Marble Corp sought to intervene in the cases alleging that the TRO
4. MINERS ASSOCIATION OF THE PHILIPPINES, INC., petitioner, vs. issued by the Court, the DENR Regional Office in San Fernando Pampanga refused to renew its Mines
HON. FULGENCIO S. FACTORAN, JR., Secretary of Environment and Natural Resources, and Temporary Permit and claimed further that its rights and interest are prejudicially affected by AO No.57
JOEL D. MUYCO, Director of Mines and Geosciences Bureau, respondents and 82.

Issue:
G.R. No. 98332 January 16, 1995 ROMERO, J.:
1. Whether AO57 and 82 have the effect of abrogating the existing mining laws and unduly preterminate
Facts: the existing mining leases and agreements.

Former President Cory Aquino, exercising legislative power, issued EO211 prescribing the interim Held: No.
procedures in the processing and approval of exploration, development and utilization of minerals. To PD No. 463, as amended, pertains to the old system of EDU of natural resources through license,
implement the act, Secretary of DENR promulgated AO No. 57 and 82. On 25 July 1987 the then
concession or lease which has been disallowed by Article XII, Sec 2 of the 1987 Philippine Constitution.
President Aquino issued EO279 authorizing the DENR SECRETARY to negotiate and conclude joint By virtue of this constitutional mandate and its implementing laws, the provisions dealing with license,
venture, co-production, production sharing, development and those agreements involving technical or
concession or leases ceased to operate as the governing laws. In other words, in all areas of
financial assistance by foreign owned corporations for large scale EDU. administration and management of mineral lands, the provision of PD463, as amended and other existing
Pursuant to Sec 6 of EO 279, the DENR issued AO No.57 which provides that all existing mining leases laws still govern.
or agreements which were granted AFTER the affectivity of the 1987 Constitution, except small scale
Upon the effectivity of the 1987 Consti on 2 February 1987, the State assumed a more dynamic role in
mining leases and those pertaining to sand and gravel and quarry resources covering an area of 20 EDU. Article XII, Section 2 explicitly ordains that EDU shall under the full control and supervision of
hectares or less, shall be converted into production sharing agreement within one year from the the State. Given these considerations, there is no clear showing that the DENR Sec has transcended the
effectivity of the guidelines.
bounds demarcated in the EO279 for the exercise of his rule-making power.
On 20 November 1990, Sec of DENR issued EO No. 82 laying down the Procedural Guidelines on the
Article XII, Sec 2 of the 1987 Consti does not apply retroactively to license, concession or lease granted
award of mineral Production sharing agreement. This order provides the person or entities required to by the government under the 1973 Consti or before the effectivity of the present Constitution. The intent
submit a LETTER OF INTENT and MINERAL PRODUCTION SHARING AGREEMENT within 2
to apply prospectively was stressed during the deliberations in the Constitutional Commission.
years from the effectivity of AO No.57 or until 17 July 1991. Failure to do so within the prescribed
period shall cause the abandonment of mining, quarry, gravel and sand. AO No. 57 applies only to all existing mining leases or agreements which were granted after the
effectivity of the 1987 Consti pursuant to EO No. 211. It bears to mention that under the text of EO211,
The Miners Association Inc assailed the validity of the above-mentioned issuances and alleged the
there is a reservation clause which provides that the privileges as well as the terms and conditions of all
following: existing mining leases or agreements granted after the effectiviyt of the present constitution shall be
1. Sec of DENR issued both AO57 and 82 in excess of their rule-making power under Section 6 of subject to any and all modification or alterations which the Congress may adopt. Hence, the strictures of
EO279. the non-impairment of contract clause do not apply to the aforesaid mining leased or agreements after the
effectivity of the 1987 Consti.
2. The orders violate the non-impairment of contracts provision under the bill of rights on the ground the
AO57 unduly pre-terminates existing mining leases and other mining agreements and automatically The State in the exercise of police power may not be precluded by the constitutional restriction on non-
converts them into production-sharing agreements within one year from its effectivity date. impairment of contracts. Police power being co-extensive with the necessities of the case and the
demands of public interest.

34
5. REPUBLIC V. ROSEMOOR MINING Tbe Petition is GRANTED.

FACTS: SC set aside the decision of the CA and TC.

Four respondents were granted permission to look for marble deposits in the mountains of Biak-na-Bato. 1. THE LICENSE IS NOT VALID.
When they discovered marble deposits in Mount Mabio, they applied for license to exploit said marble
deposits and they were issued such license. However, in a letter, Ernest o Maceda (newly-appointed The issue has not been mooted because while RA 7942 has expressly repealed provisions of mining laws
Minister of the Department of Energy and Natural Resources) cancelled their license. Prclamation No. that are inconsistent with its own, it respects previously issued valid and existing licenses.
84 was then issued, confirming the cancellation of the license.
When the license was issued, the governing law was PD 463. Thus, it was subject to the terms and
RULING OF THE TRIAL COURT: conditions of PD 463, including the part where it says that the quarry license shall cover an area of not
more than 100 hectares in any one province and not more than 1000 hectares in the entire Philippines.
The privilege granted under respondents’ license already became a property right, which is protected The license in question was issued in the name of Rosemoor Mining Development Corporation and not
unde the due process clause. License cancellation, without notice and hearing was unjust. Moreover, the the 4 individual stockholders. It clearly violates PD 463 because the license covered an area of 330-
proclamation, which confirmed the cancellation of the license was an ex post facto law. hectares.
RULING OF THE COURT OF APPEALS: 2. PROCLAMATION NO. 84, CONFIRMING THE CANCELLATION OF THE LICENSE, IS
CA affirmed the decision of the Trial Court. VALID.

It adds that the Constitution provides for the non-impairment of obligations and contracts, which implies Respondents’ license may be revoked or rescinded by executive action when the national interest so
that the license of the respondents must be respected. requires because it is not a contract, property or a property right protected by the due process clause of
the Constitution. The license itself provides such condition. The license can also be validly revoked by
ISSUES: the State in the exercise of police power, in accordance with the Regalian doctrine.
1. WON the license was valid. Also, since the license is not a contract, the non-impairment clause may not be invoked. Even if it were,
the non-impairment clause must yield to the police power of the State.
Petitioners: License was issued in violation of PD 463 – a quarry license should cover not more
than 100 hectares in any given province. The license was issued to Rosemoor Mining and The proclamation cannot also be said to be a bill-of-attainder, which is a legislative act which inflicts
Development Corporation and covered a 330-hectare land. punishment without judicial trial. The proclamation only declares the nullity of the license. It does not
declare guilt or impose punishment.
Respondents: The license was embraced by 4 separate applications, each for an aread of 81
hectares. Also, the issue has been mooted because PD 463 has already been repealed by RA The proclamation can also be said to be an ex post facto law because it does not fall under any of the six
7942 or the Philippine Mining Act of 1995. recognized instances when a law is considered as such. It is not even criminal or penal in nature.

2. WON Proclamation No. 84 – which confirmed the cancellation of the license, is valid. Lastly, when President Aquino issued Proclamation No. 84, she was still validly exercising legislative
powers under the Provisional Constitution of 1986.
Petitioners: The license was validly declared a nullity and terminated. Maceda’sletter did not cancel or
revoke the license, but merely declared its nullity. Also, the respondents waived their right to notice and 6. YINLU BICOL MINING CORPORATION, Petitioner, v. TRANS-ASIA OIL AND ENERGY
hearing in their license application. DEVELOPMENT CORPORATION, Respondent.

Respondents: Their right to due process was violated because there was no notice and hearing. [G.R. No. 207942, January 12, 2015, BERSAMIN, J.:]
Proclamation No. 84 is not valid because it violates the clause on non-impairment of contracts, it is an ex
TOPIC: PERSONS – EFFECT AND APPLICATION OF LAWS – REPEAL OF LAWS
post facto law and/or a bill of attainder, and it was issued by the President after the effectivity of the
1987 Constitution. DOCTRINE: Rights pertaining to mining patents issued pursuant to the Philippine Bill of 1902 and
existing prior to November 15, 1935 are vested rights that cannot be impaired.
RULING OF THE SUPREME COURT:
35
FACTS: b. He rejected Trans-Asia’s argument that Yinlu’s patents had no effect and were deemed
abandoned because Yinlu had failed to register them pursuant to Section 101 of
1. This case involves 13 mining claims over the area located in Barrio Larap, Municipality of Jose Presidential Decree No. 463, as amended.
Panganiban, Camarines Norte, a portion of which was owned and mined by Philippine Iron c. He refuted Trans-Asia’s contention that there was a continuing requirement
Mines, Inc. (PIMI), which ceased operations in 1975 due to financial losses. under the Philippine Bill of 1902 for the mining patent holder to undertake
a. PIMI’s portion (known as the PIMI Larap Mines) was sold in a foreclosure sale to the improvements in order to have the patents subsist, and that Yinlu failed to
Manila Banking Corporation (MBC) and Philippine Commercial and Industrial Bank perform its obligation to register and to undertake the improvement, observing
(PCIB, later Banco De Oro, or BDO). that the requirement was not an absolute imposition.
2. The Government then opened the area for exploration. i. He noted that the suspension of PIMI’s operation in 1974 due to financial
a. Trans-Asia Oil and Energy Development Corporation (Trans-Asia) then explored the losses and the foreclosure of its mortgaged properties by the creditor banks
area from 1986 onwards. (MBC/PCIB) constituted force majeure that justified PIMI’s failure in 1974
i. In 1996, it entered into an operating agreement with Philex Mining to comply with the registration requirement under P.D. No. 463;
Corporation over the area, their agreement being duly registered by the ii. that the Philippine Bill of 1902, which was the basis for issuing the patents,
Mining Recorder Section of Regional Office No. V of the Department of allowed the private ownership of minerals, rendering the minerals covered by
Environment and Natural Resources (DENR). the patents to be segregated from the public domain and be considered private
3. 1997: Trans-Asia filed an application for the approval of Mineral Production Sharing property; and
Agreement (MPSA) over the area in that Regional Office of the DENR, through the Mines and iii. that the Regalian doctrine, under which the State owned all natural resources,
Geosciences Bureau (MGB), in Daraga, Albay. was adopted only by the 1935, 1973 and 1987 Constitutions
a. The application, which was amended in 1999, was granted on July 28, 2007 under 9. Office of the President affirmed the DENR Sec’s Order.
MPSA No. 252-2007-V, by which Trans-Asia was given the exclusive right to a. Under the Philippine Constitution, there is an absolute prohibition against alienation of
explore, develop and utilize the mineral deposits in the portion of the mineral natural resources. Mining locations may only be subject to concession or lease.
lands. i. The only exception is where a location of a mining claim was perfected prior
4. August 31 2007: Yinlu Bicol Mining Corporation (Yinlu) informed the DENR by letter to November 15, 1935, when the government under the 1935 Constitution
that it had acquired the mining patents of PIMI from MBC/BDO by way of a deed of was inaugurated, and according to the laws existing at that time a valid
absolute sale, stating that the areas covered by its mining patents were within the areas of location of a mining claim segregated the area from the public domain, and
Trans-Asia’s MPSA the locator is entitled to a grant of the beneficial ownership of the claim and
5. September 14, 2007: Trans-Asia informed Yinlu by letter that it would commence exploration the right to a patent therefore.
works in Yinlu’s areas pursuant to the MPSA, and requested Yinlu to allow its personnel to b. The right of the locator to the mining patent is a vested right, and the
access the areas for the works to be undertaken. Constitution recognizes such right as an exception to the prohibition against
a. Yinlu replied that Trans-Asia could proceed with its exploration works on its own alienation of natural resources.
private property in the Calambayungan area, not in the areas covered by its (Yinlu) i. The right of the appellee as the beneficial owner of the subject mining patents
mining patents. in this case, therefore, is superior to the claims of appellant
6. TransAsia found out that the registration of its MPSA had been put on hold because of ii. The existence of the TCT’s in the name of appellee further bolsters the
Yinlu’s request to register the deed of absolute sale in its favor. existence of the mining patents. Under PD 1529, also known as the Property
7. DENR Secretary directed MGB Regional Office V to verify the validity of the mining patents Registration Decree, once a title is cleared of all claims or where none exists,
of Yinlu. the ownership over the real property covered by the Torrens title becomes
a. MGB Regional Office V informed the Office of the DENR Secretary that there was no conclusive and indefeasible even as against the government.
record on file showing the existence of the mining patents of Yinlu. Accordingly, the 10. CA: It agreed with the DENR Secretary and the OP that Yinlu held mining patents over the
parties were required to submit their respective position papers disputed mining areas, but ruled that Yinlu was required to register the patents under PD
8. DENR Sec Atienza ordered the amendment of Trans-Asia’s MPSA by excluding therefrom the No. 463 in order for the patents to be recognized in its favor.
mineral lands covered by Yinlu’s mining patents a. It found that Yinlu and its predecessors-in-interest did not register the patents pursuant
a. DENR Sec. Jose L. Atienza, Jr in his order found that the mining patents had been to PD No. 463; hence, the patents lapsed and had no more effect
issued to PIMI in 1930 as evidenced by and indicated in PIMI’s certificates of title 11. Yinlu asserts the following:
submitted by Yinlu; and that the patents were validly transferred to and were now a. The mining patents of Yinlu were registered pursuant to Act No. 496 (Land
owned by Yinlu. Registration Act of 1902) in relation to the Philippine Bill of 1902 (Act of Congress of

36
July 1 , 1902), the governing law on the registration of mineral patents, were valid, 3. Even without a patent, the possessory right of a qualified locator after discovery of minerals
existing and indefeasible. upon the claim is a property right in the fullest sense, unaffected by the fact that the paramount
i. Section 21 of the Philippine Bill of 1902: allowed citizens of the United title to the land is in the Government, and it is capable of transfer by conveyance, inheritance, or
States and of the Philippine Islands to explore, occupy and purchase mineral devise.
lands a. the mining claim under consideration no longer formed part of the public domain
ii. Section 27 of the Philippine Bill of 1902: after the exploration and claim of when the provisions of Article XII of the Constitution became effective, it does not
the mineral land, the owner of the claim and of the mineral patents was come within the prohibition against the alienation of natural resources; and the
entitled to all the minerals found in the area subject of the claim petitioner has the right to a patent therefor upon compliance with the terms and
iii. its registered mineral patents, being valid and existing, could not be defeated conditions prescribed by law.
by adverse, open and notorious possession and prescription; 4. Although Section 100 and Section 101 of PD No. 463 require registration and annual work
b. substantive rights over mineral claims perfected under the Philippine Bill of 1902 obligations, Section 99 of PD No. 463 nevertheless expressly provides that the provisions of PD
subsisted despite the changes of the Philippine Constitution and of the mining No. 463 shall not apply if their application will impair vested rights under other mining laws
laws Section 99. Non-impairment of Vested or Acquired Substantive Rights. Changes made and new
i. Constitution could not impair vested rights; provisions and rules laid down by this Decree which may prejudice or impair vested or acquired rights in
ii. Section 100 and Section 101 of PD No. 463 would impair its vested rights accordance with order mining laws previously in force shall have no retroactive effect. Provided, That
under its mineral patents if said provisions were applied to it; the provisions of this Decree which are procedural in nature shall prevail.
iii. Section 99 of PD No. 463 expressly prohibited the application of Section
100 and Section 101 to vested rights. a. A right is vested when the right to enjoyment has become the property of some
particular person or persons as a present interest.
i. It is “the privilege to enjoy property legally vested, to enforce contracts, and
ISSUE: Whether Yinlu’s mining patents constitute vested rights and could not be disregarded.
enjoy the rights of property conferred by existing law” or “some right or
HELD: YES interest in property which has become fixed and established and is no longer
open to doubt or controversy”
1. A mining patent pertains to a title granted by the government for the said mining claim. b. The due process clause prohibits the annihilation of vested rights. ‘A state may not impair
2. Under the 1935 Constitution, which took effect on November 15 1935, the alienation of natural vested rights by legislative enactment, by the enactment or by the subsequent repeal of a
resources, with the exception of public agricultural land, was expressly prohibited. municipal ordinance, or by a change in the constitution of the State, except in a legitimate
a. The natural resources being referred therein included mineral lands of public domain, exercise of the police power’
but not mineral lands that at the time the 1935 Constitution took effect no longer c. It has been observed that, generally, the term “vested right” expresses the concept of
formed part of the public domain. present fixed interest, which in right reason and natural justice should be protected against
b. Prohibition against the alienation of natural resources did not apply to a mining arbitrary State action, or an innately just an imperative right which an
claim or patent existing prior to November 15, 1935. enlightened free society, sensitive to inherent and irrefragable individual rights, cannot
c. McDaniel v. Apacible: A mining claim perfected under the law is property in the deny
highest sense, which may be sold and conveyed and will pass by descent. It has the d. Republic v. Court of Appeals: that mining rights acquired under the Philippine Bill of 1902
effect of a grant (patent) by the United States of the right of present and exclusive and prior to the effectivity of the 1935 Constitution were vested rights that could not be
possession of the lands located. impaired even by the Government.
d. The owner of a perfected valid appropriation of public mineral lands is entitled to the e. In the present case: the mining patents of Yinlu were issued pursuant to the Philippine
exclusive possession and enjoyment against everyone, including the Government itself. Bill of 1902 and were subsisting prior to the effectivity of the 1935 Constitution.
Where there is a valid and perfected location of a mining claim, the area becomes Consequently, Yinlu and its predecessors-in-interest had acquired vested rights in the
segregated from the public domain and the property of the locator. disputed mineral lands that could not and should not be impaired even in light of their past
e. A valid and subsisting location of mineral land, made and kept up in accordance with failure to comply with the requirement of registration and annual work obligations.
the provisions of the statutes of the United States, has the effect of a grant by the
United States of the present and exclusive possession of the lands located, and this
exclusive right of possession and enjoyment continues during the entire life of the G.R. No. L-49143 August 21, 1989
location. There is no provision for, nor suggestion of, a prior termination thereof.

37
7. ZAMBALES CHROMITE MINING COMPANY, INC., petitioner, On May 10, 1979, petitioner filed a reply (Rollo, p. 83) to the comment in compliance with the resolution
vs. of April 10, 1979. But on May 9, 1979, Baguio Gold Mining Company, Philex Mining and Regalian
HON. MINISTER OF NATURAL RESOURCES JOSE J. LEIDO JR. and DIRECTOR OF Mining Corporation filed with the Court two separate motions for leave to intervene (Rollo, p. 120).
MINES JUANITO C. FERNANDEZ, respondents. PHILEX MINING CORPORATION and
REGALIAN MINING EXPLORATION CORPORATION, intervenors. On February 10, 1981, Baguio Gold Mining Company, Philex Mining Company and Regalian Mining
Corporation filed with the court a Joint Petition for Intervention (Rollo, p. 171) raising the same issues
This is a petition for certiorari and prohibition with preliminary injunction seeking to enjoin the Minister brought up by petitioner Zambales Chromite Mining Company regarding the constitutionality of P.D. No.
(now Secretary) of Natural Resources and the Director of Mines from enforcing Presidential Decree No. 1214 based on the doctrinal mandates of the ruling cases of McDaniel v. Apacible, 42 Phil. 749 [1922]
1214 dated October 14,1977 requiring all locators under the Act of Congress of July 1, 1902, as amended, and Gold Creek Mining Corporation v. Rodriguez, 66 Phil. 259 (1939); Salazar Mining Co. v. Rodriguez,
to apply for mininglease contracts under the provision of Presidential Decree No. 463 better known as the et al., 67 Phil. 97, insofar as it invests inter alia, private ownership in patentable mining claims to have
Mineral Development Resources Decree of 1974 and to declare Presidential Decree No. 1214 survived to date due to a faithful compliance with the various requirements of applicable mining laws to
unconstitutional since its enforcement would deprive petitioners of its property without due process and include the land surface of said mining claims. Petitionees memorandum was adopted by intervenors as to
without just compensation. the factual and legal showing of the unconstitutionality of Presidential Decree No. 1214 (Rollo, pp. 455-
456).
Petitioner Zambales Chromite Mining Company, Inc. is a mining corporation duly organized and existing
under and by virtue of the laws of the Philippines. The Solicitor General as counsel for public respondent submitted his memorandum on February 12,1982
(Rollo, pp. 468499) while petitioner filed its reply to said memorandum on April 3, 1982 (Rollo, pp. 505-
560).
Petitioner claims that it is the owner and holder of sixty (60) mineral claims which it acquired through
purchase in good faith and for value 43 years ago. Said claims situated at the Municipality of Sta. Cruz,
Zambales, were located and registered in 1934 under the Act of U.S. Congress of July 1, 1902 (known as Counsel for petitioner on August 20, 1982 filed a motion to refer this case to the Court En Bane for action
the Philippine Bill of 1902). (Petition, p. 2; Rollo, p. 3); that from 1934 to 1977 it has to its credit a total and decision (Rollo, p. 536) and on September 8,1982, the Court resolved to issue a temporary restraining
investment of over Pl,222,640.00 for the mining exploration, development and operation of its said sixty order, effective as of said date and continuing until otherwise ordered by the Court (Rollo, p. 562).
mining claims (Petition, p. 3; Rollo, p. 4); that on June 14, 1977 it actually and duly flied its application
for patent for each claim of said sixty (60) mineral claims (Petition, p. 4; Rollo, p. 5); that respondent On February 11, 1988 the Court acting on the motion for intervention filed by counsel for intervenor
Director of Mines issued an order dated July 13,1977 approving the application of petitioner for availment Francisco N. Calinisan dated January 6,1988, and considering that this case has long been submitted for
decision, resolved to deny the aforesaid motion for having been filed late (Reno, p. 597).
of rights on said claims under Presidential Decree No. 463 (Petition, p. 5; Rollo, p. 6); that the aforesaid
sixty (60) lode mineral claims are already private property of petitioner, following the doctrinal rule laid
down in McDaniel v. Apacible and Cuisia (42 Phil. 749; 753-754) andGold Creek Mining Corporation v. The principal issue raised by the petitioner and by the erstwhile intervenors, is: whether or not under the
Rodriguez, et al. (66 Phil. 259) which had already been segregated from the public domain to which provision of P.D. No. 1214 there was deprivation of property without due process of law and just
petitioner is entitled to the exclusive possession and enjoyment against everyone; that the issuance of compensation which makes said decree unconstitutional.
Presidential Decree No. 1214 on October 14, 1977 which declared open to lease subsisting and valid
patentable mining claims, lode or placer, located under the provisions of the Act of U.S. Congress of July Their contention that a perfected and valid appropriation of public mineral lands operates as a withdrawal
1, 1902, as amended, already segregated from the public domain and owned and held by it for over 43 of the tract of land from the public domain and is deemed to be already private property, is without basis
years and requiring it without fail and against their will to file a mining lease application with the in fact and in law (Comment, Rollo, p. 61)
Mines Regional Office concerned within a period of one year from October 14, 1977 is a deprivation of
petitioner's rights to the ownership of said claims without due process of law nor or just compensation and This issue has been resolved in a recent decision of this Court in Sta. Rosa Mining Co., Inc. vs. Leido
therefore, unconstitutional. Jr. (156 SCRA 1 [1987]) where it was held that while rulings in McDaniel v. Apacible (42 Phil. 749
[1922]). and Gold Creek Mining Corp. v. Rodriguez (66 Phil. 259 [1938]) cited by the petitioner, true
The Court in its resolution dated November 3,1978, gave due course to the petition and required enough, recognize the right of a locator of a mining claim as a property right; such right is not absolute. It
respondents to comment (Rollo, p. 33).lâwphî1.ñèt The Solicitor General as counsel for public respondent, is merely a possessory right more so if petitioner's claims are still unpatented. It can be lost through
flied his comment on March 26,1979 (Rollo, pp. 58-71-A). abandonment or forfeiture or they may be revoked on valid legal grounds.

38
In the case at bar, there is no showing that petitioner has complied with all the terms and conditions thereby within the period herein prescribed shall cause the forfeiture of all his rights to the claim." (Director
prescribed by law prior to November 1, 1935; that there should be not only a valid and subsisting location of Lands v. Kalahi Investments, Inc., G.R. No. L-48066, January 31, 1989). (Emphasis supplied)
of the mineral land but that there should be, thereafter, continuous compliance with all the requirements
of law such as the performance of annual assessment works and payment of real estate taxes. In fact, PREMISES CONSIDERED, the instant petition is DENIED for lack of merit.
petitioner filed its application only in 1977 for a patent, or 43 years after it allegedly located and registered
the mining claims (Rollo, p. 63).lâwphî1.ñèt SO ORDERED

As to the issue of constitutionality, the Court categorically stated that P.D. No. 1214 is constitutional. The
8. PNOC-ENERGY DEVELOPMENT CORPORATION (PNOC-EDC), Petitioner, vs.
Court ruled:
EMILIANO G. VENERACION, JR., Respondent.
...It is a valid exercise of the sovereign power of the State, as owner, over lands of the public domain, of DECISION
which petitioner's mining claims still form a part, and over the patrimony of the nation, of which mineral
deposits are a valuable asset. It may be underscored, in this connection, that the Decree does not cover all CHICO-NAZARIO, J.:
mining claims located under the Phil. Bill of 1902, but only those claims over which their locators had
failed to obtain a patent. And even then, such locators may still avail of the renewable twenty-five year This case involves the conflicting claims of the petitioner Philippine National Oil CorporationEnergy
(25) lease prescribed by Pres. Decree No. 463, the Mineral Development Resources Decree of 1974. Development Corporation and the respondent over the mining rights over Block 159 of the Malangas
Coal Reservation, Alicia, Zamboanga del Sur.
Mere location does not mean absolute ownership over the affected land or the mining claim. It merely FACTS
segregates the located land or area from the public domain by barring other would be locators from locating
the same and appropriate for themselves the minerals found therein. To rule otherwise would imply that On 31 January 1989, respondent applied with the Mines and Geo-Sciences Development Services,
location is all that is needed to acquire and maintain rights over a located mining claim. This, we cannot DENR, Region IX, Zamboanga City for a Declaration of Location (DOL) over Block 159 of the
approve or sanction because it is contrary to the intention of the lawmaker that the locator should faithfully Malangas Coal Reservation, situated at Barangays Payongan and Kauswagan, Alicia, Zamboanga del
and consistently comply with the requirements for annual work and improvements in the located mining Sur. On 18 May 1989, the Office of the Regional Executive Director (RED) of the DENR informed the
claim. (Santa Rosa Mining Co., Inc. vs. Leido Jr., supra, pp. 8-9) respondent that his DOL cannot be registered since Block 159 was part of the Malangas Coal
Reservation, as provided under Proclamation No. 284, issued by the President on 19 July 1938. With the
P.D. No. 1214 is in accord with Section 8, Article XIV of the 1973 Constitution and presently in Section endorsement of the Office of Energy Affairs (OEA) and the DENR Secretary, the respondent petitioned
2, Article XII of the 1987 Constitution where the same constitutional mandate is restated.
the Office of the President for the withdrawal of Block 159 from the coal reservation and its conversion
into a mineral reservation.
On June 2,1988, the Court granted a motion filed by counsel for petitioner dated May 20,1988 to admit a
manifestation and motion wherein petitioner prayed that the "Court allow the petitioner to change the The petitioner applied for a mineral prospecting permit over Block 159 (and Blocks 120 and 160) with
original prayer in its petition dated October 10, 1978 with a new prayer directing public respondents to the OEA, which the latter granted on 4 September 1989. The Malangas Coal Reservation was, at that
dispose of petitioner's application on its own merit unaffected and without regard to the provision of P.D. time, under the administration of the OEA. When it had initially applied for a mineral prospecting permit
1214 . . ." (p. 631, Rollo) over lands within the Malangas Coal Reservation, the OEA advised it to obtain the permission of the
Bureau of Mines and GeoSciences (BMGS). On 18 October 1991, petitioner submitted to the DENR an
Records show that petitioner Zambales Chromite filed its patent application over its 60 mining claims on application/proposal for a Mineral Production Sharing Agreement (MPSA) over Blocks 120, 159 and
June 14,1977 and to order such disposal of said "application on its own merit" is not within the scope of 160 of the Malangas Coal Reservation. On 21 February 1992, the Officer-In-Charge Regional Technical
the jurisdiction of the Court. For, even assuming claimant to be a holder of a subsisting and valid patentable Director Dario R. Miñoza of the Mines and Geo-Sciences Developmental Service (MGDS) advised the
mining claim, this Court has held that it can no longer proceed with the acquisition of a mining patent in
petitioner to amend its
view of P.D. No. 1214, issued on October 14, 1977, directing holder of subsisting and patentable mining
claims, lode or placer, located under the provisions of the Act of Congress on July 1, 1902, as amended, to application for MPSA by excluding Block 159 as the same is covered by the application of the
file a mining lease application . . . within one year from the approval of the Decree and upon the filing respondent. Nevertheless, the petitioner did not exclude Block 159 from its MPSA. Records also show
thereof, holders of said claims shall be considered to have waived their rights to the issuance of mining
that it had not applied for nor was it able to obtain an Exploration Permit from the BMGS over Block
patents therefor: Provided, however, that the non-filing of the application for mining lease by the holders
159.

39
The MAB also decreed that the respondent had preferential mining rights over Block 159. It ruled that
the proper procedure with respect to the mining rights application over Block 159 when it was still part
On 13 April 1992, Presidential Proclamation No. 890 was issued, which effectively excluded Block 159 of the Malangas Coal Reservation required the following: (1) application for prospecting permit with the
from the operation of Proclamation No. 284, and declared Block No. 159 as government mineral OEA or other office having jurisdiction over said reservation; (2) application for exploration permit; (3)
reservation open for disposition to qualified mining applicants, pursuant to Executive Order No. 279. application for exclusion of the land from such reservation; (4) Presidential Declaration on exclusion as
On 26 May 1992, petitioner’s application for MPSA covering Coal Block Nos. 120, 159 and 160 was recommended by the Secretary; and (5) application for Lease thereof with priority given to holder of
accepted for filing. Respondent immediately filed, on 28 May 1992, a protest to the petitioner’s inclusion exploration Permit.
of Block 159 in its application for MPSA before the RED of the DENR Office in Zamboanga City.
After the parties were heard, the RED, in an Order, dated 12 April 1993, ruled in favor of the respondent The MAB noted that petitioner did not file for an exploration permit nor applied for the exclusion of
and ordered the petitioner to amend its MPSA by excluding therefrom Block 159. On 18 May 1993, Block 159. Moreover, petitioner filed a MPSA on
petitioner filed a Motion for Reconsideration of the Order dated 12 April 1993, which the RED denied in
an Order dated 5 July 1993. On 30 July 1993, petitioner filed an appeal with the DENR Secretary 18 October 1991, or almost six (6) months prior to the issuance of Proclamation No. 890 excluding
questioning the Orders issued by the RED. Block 159 from the Malangas Coal Reservation and allowing its disposition. Thus, the application for a
MPSA over Block 159, while it was still part of a government reservation other than a mineral
While the case was pending, respondent applied for a MPSA. On 31 July 1992, he paid the processing reservation, was erroneous and improper and could not have been legally accepted. And, since the
fee for a MPSA covering Block 159 and was able to comply with all other requirements of the MPSA records show that only one MPSA was filed after the issuance of Proclamation 890 – that of the
application. On 4 October 1994, the Office of the Secretary dismissed the appeal on the ground that respondent’s, the preferential right over Block 159 was acquired by the respondent. The MAB,
petitioner’s right to appeal had already prescribed. Section 50 of Presidential Decree No. 463 provides nevertheless, pointed out that the said preferential right does not necessarily lead to the granting of the
therefore for a five-day reglementary period from the receipt of the order or decision of the Director. respondent’s MPSA, but merely consists of the right to have his application evaluated and the
Petitioner received its copy of the assailed Order dated 12 April 1993 on 7 May 1993, but filed its prohibition against accepting other mining applications over Block 159 pending the processing of his
Motion for Reconsideration only on 18 May 1993, or eleven days after its receipt thereof. Thereafter, MPSA.
petitioner received a copy of the Order dated 5 July 1993 on 16 July 1993, but filed its appeal only on 30
July 1993 or nine days after the allowable period to appeal. ISSUES:

On 25 October 1994, petitioner, through a letter addressed to the DENR Secretary, sought the There are two main issues that need to be resolved in this case: (1) whether or not the petitioner has
reconsideration of the Decision, dated 4 October 1994. In a Resolution, dated 21 December 1994, the already lost its right to appeal the RED’s Order dated 12 April 1993; and (2) whether or not the
then DENR Secretary Angel C. Alcala reversed the Decision, dated 4 October 1994, and gave due course petitioner acquired a preferential right on mining rights over Block 159.
to the MPSA of the petitioner. HELD: (kopyahon ko na lang ang sa SCRA)
On 1 February 1995, respondent filed a Motion for Reconsideration of the Resolution, dated 21 Administrative Law; Appeals; Appeals from judgments and final orders of quasi-judicial bodies are
December 1994. The now DENR Secretary Victor O. Ramos issued an Order, dated 5 August 1996, required to be brought to the CA, under the requirements and conditions set forth in Rule 43 of the Rules
reversing the Resolution, dated 21 December 1994 and reinstating the Decision, dated 4 October 1994. It of CivPro. With the enactment of Republic Act No. 7902, this Court issued Circular No. 1-95 dated 16
ruled that the Orders issued by the RED have already become final and executory when the petitioner May 1995 governing appeals from all quasi-judicial bodies to the Court of Appeals by petition for
failed to file its appeal five days after it had received the Orders. As a result, the DENR Secretary no review, regardless of the nature of the question raised. Said circular was incorporated in Rule 43 of the
longer had the jurisdiction to issue the assailed Resolution, dated 21 December 1994. It added that after Rules of Civil Procedure. In addition, this Court held in a line of cases that appeals from judgments and
looking into the merits of the case, the Orders of the RED were in accordance with the evidence on final orders of quasi-judicial bodies are required to be brought to the Court of Appeals, under the
record and the pertinent laws on the matter. On 20 August 1996, petitioner filed a Motion for requirements and conditions set forth in Rule 43 of the Rules of Civil Procedure. Nevertheless, this Court
Reconsideration of the Order, dated 5 August 1996. On 21 May 1997, the MAB resolved the motion in has taken into account the fact that these cases were promulgated after the petitioner filed this appeal on
favor of the respondent and affirmed the assailed Order, dated 5 August 1996. It took cognizance of the 4 August 1997, and decided to take cognizance of the present case.
appeal filed by petitioner, in accordance with Section 78 of Republic Act No 7942, otherwise known as
The Philippine Mining Act of 1995. The MAB ruled that the petitioner filed its appeal beyond the five-
day prescriptive period provided under Presidential Decree No. 463, which was then the governing law
on the matter.

40
Natural resurces; mines, by providing a five-day period within which to file an appeal on the decisions of Mining Location. – No prospecting and exploration shall be allowed: (a) In military, and other
the Dir. Of Mines and Geo-Sciences, PD 463 unquestionably repealed Section 61 of CA 137 - When Government reservations except when authorized by the proper Government agency concerned. Section
Presidential Decree No. 463 was enacted in 1974, Section 50 of the law had clearly intended to repeal 8 of Presidential Decree No. 463 reiterates the rule and clarifies it further by stating that prospecting,
the corresponding provision found in Section 61 of Commonwealth Act No. 137, and to shorten the 30- exploration and exploitation of minerals on reserved lands other than mineral reservations may be
day period within which to file an appeal from the Decision of the Director of Mines and Geo-Sciences undertaken by the proper government agency. As an exception to this rule, qualified persons may
to five days. Section 61 of Commonwealth Act No. 137, as amended, provides that: SEC. 61. - Conflicts undertake the said prospecting, exploration and exploitation when the said agencies cannot undertake
and disputes arising out of mining locations shall be submitted to the Director of Mines for decision: them.
Provided, That the decision or order of the Director of Mines may be appealed to the Secretary of
Agriculture and Natural Resources within thirty days from receipt of such decision or order. In case any Petition denied!
one of the parties should disagree from the decision or order of the Secretary of Agriculture and Natural G.R. No. 195580 April 21, 2014
Resources, the matter may be taken to the Court of Appeals or the Supreme Court, as the case may be,
within thirty days from the receipt of such decision or order, otherwise the said decision or order shall be
9. NARRA NICKEL MINING AND DEVELOPMENT CORP., TESORO MINING AND
final and binding upon the parties concerned. x x x. Section 50 of Presidential Decree No. 463 reads:
DEVELOPMENT, INC., and MCARTHUR MINING, INC., Petitioners, 
vs.
REDMONT
Sec. 50. Appeals. - Any party not satisfied with the decision or order of the Director, may, within five (5)
CONSOLIDATED MINES CORP., Respondent.
days from receipt thereof, appeal to the Minister [now Secretary]. Decisions of the Minister [now
Secretary] are likewise appealable within five (5) days from receipt thereof by the affected party to the FACTS:
President whose decision shall be final and executory.
Sometime in December 2006, respondent Redmont Consolidated Mines Corp. (Redmont), a domestic
Petitioner’s insistence that the 30-day reglementary period provided by Section 61 of Commonwealth corporation organized and existing under Philippine laws, took interest in mining and exploring certain
Act No. 137, as amended, applies, cannot be sustained by this Court. By providing a five-day period areas of the province of Palawan. After inquiring with the Department of Environment and Natural
within which to file an appeal on the decisions of the Director of Mines and GeoSciences, Presidential Resources (DENR), it learned that the areas where it wanted to undertake exploration and mining activities
Decree No. 463 unquestionably repealed Section 61 of Commonwealth Act No. 137. where already covered by Mineral Production Sharing Agreement (MPSA) applications of petitioners
Narra, Tesoro and McArthur.
Rules of Procedure must be faithfully followed except only when for persuasive reasons, they may be
relaxed to relieve a litigant of an injustice not commensurate with his failure to comply with the Petitioner McArthur Narra and Tesoro, filed an application for an MPSA and Exploration Permit (EP)
prescribed procedure - Petitioner invokes the judicial policy of allowing appeals, although filed late, which was subsequently issued.
when the interest of justice so requires. Procedural law has its own rationale in the orderly administration
of justice, namely, to ensure the effective enforcement of substantive rights by providing for a system On January 2, 2007, Redmont filed before the Panel of Arbitrators (POA) of the DENR three (3) separate
that petitions for the denial of petitioners’ applications for MPSA.

obviates arbitrariness, caprice, despotism, or whimsicality in the settlement of disputes. Hence, rules of Redmont alleged that at least 60% of the capital stock of McArthur, Tesoro and Narra are owned and
procedure must be faithfully followed except only when for persuasive reasons, they may be relaxed to controlled by MBMI Resources, Inc. (MBMI), a 100% Canadian corporation. Redmont reasoned that since
relieve a litigant of an injustice not commensurate with his failure to comply with the prescribed MBMI is a considerable stockholder of petitioners, it was the driving force behind petitioners’ filing of
procedure. Concomitant to a liberal application of the rules of procedure should be an effort on the part the MPSAs over the areas covered by applications since it knows that it can only participate in mining
of the party invoking liberality to explain his failure to abide by the rules.30 In the instant case, petitioner activities through corporations which are deemed Filipino citizens. Redmont argued that given that
failed to state any compelling reason for not filing its appeal within the mandated period. Instead, the petitioners’ capital stocks were mostly owned by MBMI, they were likewise disqualified from engaging
records show that after failing to comply with the period within which to file their motion for in mining activities through MPSAs, which are reserved only for Filipino citizens.
reconsideration on time, they again failed to file their appeal before the Office of the DENR Secretary Petitioners averred that they were qualified persons under Section 3(aq) of Republic Act No. (RA) 7942
within the time provided by law. or the Philippine Mining Act of 1995. They stated that their nationality as applicants is immaterial because
Natural resources, mines: as a general rule, prospecting and exploration of minerals in a government they also applied for Financial or Technical Assistance Agreements (FTAA) denominated as AFTA-IVB-
reservation is prohibited under Sec. 13 of PD 463 - However, the same rule provides an exception 09 for McArthur, AFTA-IVB-08 for Tesoro and AFTA-IVB-07 for Narra, which are granted to foreign-
involving instances when the government agency concerned allows it. Section 13. Areas Closed to owned corporations. Nevertheless, they claimed that the issue on nationality should not be raised since

41
McArthur, Tesoro and Narra are in fact Philippine Nationals as 60% of their capital is owned by citizens only the number of shares corresponding to such percentage shall be counted as of Philippine
of the Philippines. nationality (GRANDFATHER RULE). Thus, if 100,000 shares are registered in the name of a
corporation or partnership at least 60% of the capital stock or capital, respectively, of which
On December 14, 2007, the POA issued a Resolution disqualifying petitioners from gaining MPSAs. The belong to Filipino citizens, all of the shares shall be recorded as owned by Filipinos. But if less
POA considered petitioners as foreign corporations being "effectively controlled" by MBMI, a 100% than 60%, or say, 50% of the capital stock or capital of the corporation or partnership,
Canadian company and declared their MPSAs null and void. respectively, belongs to Filipino citizens, only 50,000 shares shall be counted as owned by
Pending the resolution of the appeal filed by petitioners with the MAB, Redmont filed a Complaint with Filipinos and the other 50,000 shall be recorded as belonging to aliens.
the Securities and Exchange Commission (SEC), seeking the revocation of the certificates for registration The grandfather rule, petitioners reasoned, has no leg to stand on in the instant case since the definition of
of petitioners on the ground that they are foreign-owned or controlled corporations engaged in mining in a "Philippine National" under Sec. 3 of the FIA does not provide for it. They further claim that the
violation of Philippine laws. grandfather rule "has been abandoned and is no longer the applicable rule." They also opined that the last
CA found that there was doubt as to the nationality of petitioners when it realized that petitioners had a portion of Sec. 3 of the FIA admits the application of a "corporate layering" scheme of corporations.
common major investor, MBMI, a corporation composed of 100% Canadians. Pursuant to the first Petitioners claim that the clear and unambiguous wordings of the statute preclude the court from construing
sentence of paragraph 7 of Department of Justice (DOJ) Opinion No. 020, Series of 2005, adopting the it and prevent the court’s use of discretion in applying the law. They said that the plain, literal meaning of
1967 SEC Rules which implemented the requirement of the Constitution and other laws pertaining to the the statute meant the application of the control test is obligatory.
exploitation of natural resources, the CA used the "grandfather rule" to determine the nationality of SC disagreed. "Corporate layering" is admittedly allowed by the FIA; but if it is used to circumvent the
petitioners. Constitution and pertinent laws, then it becomes illegal. Further, the pronouncement of petitioners that the
grandfather rule has already been abandoned must be discredited for lack of basis.
In determining the nationality of petitioners, the CA looked into their corporate structures and their
corresponding common shareholders. Using the grandfather rule, the CA discovered that MBMI in Petitioners McArthur, Tesoro and Narra are not Filipino since MBMI, a 100% Canadian corporation, owns
effect owned majority of the common stocks of the petitioners as well as at least 60% equity interest 60% or more of their equity interests. Such conclusion is derived from grandfathering petitioners’
of other majority shareholders of petitioners through joint venture agreements. The CA found that corporate owners, namely: MMI, SMMI and PLMDC. The "control test" is still the prevailing mode of
through a "web of corporate layering, it is clear that one common controlling investor in all mining determining whether or not a corporation is a Filipino corporation, within the ambit of Sec. 2, Art. II of
corporations involved x x x is MBMI." Thus, it concluded that petitioners McArthur, Tesoro and the 1987 Constitution, entitled to undertake the exploration, development and utilization of the natural
Narra are also in partnership with, or privies-in-interest of, MBMI. resources of the Philippines. When in the mind of the Court there is doubt, based on the attendant facts
and circumstances of the case, in the 60-40 Filipino-equity ownership in the corporation, then it may apply
ISSUE: the "grandfather rule."
Whether or not the Court of Appeals’ ruling that Narra, Tesoro and McArthur are foreign corporations 10. DIDIPIO V. GOZUN:*
based on the "Grandfather Rule" is contrary to law, particularly the express mandate of the Foreign
Investments Act of 1991, as amended, and the FIA Rules. Constitutionality of the Mining Act:

HELD: The Facts


On June 20, 1994, before the passage of Republic Act 7042 (The Philippine Mining Act of 1995),
No. There are two acknowledged tests in determining the nationality of a corporation: the control test and President Fidel V. Ramos executed a Financial and Technical Assistance Agreement (FTAA) with
the grandfather rule. Paragraph 7 of DOJ Opinion No. 020, Series of 2005, adopting the 1967 SEC Rules Arimco Mining Corporation (AMC), over a total land area of 37,000 hectares covering the provinces of
which implemented the requirement of the Constitution and other laws pertaining to the controlling Nueva Vizcaya and Quirino. Included in that area was Barangay Didipio, Kasibu, Nueva Vizcaya.
interests in enterprises engaged in the exploitation of natural resources owned by Filipino citizens,
provides: Subsequently, AMC consolidated with Climax Mining Limited to form a single company that did
business under the new name of Climax-Arimco Mining Corporation (CAMC), of which the controlling
99 percent of stockholders were Australian nationals.
Shares belonging to corporations or partnerships at least 60% of the capital of which is
owned by Filipino citizens shall be considered as of Philippine nationality (CONTROL TEST), On September 7, 2001, counsels for petitioners filed a demand letter addressed to then Secretary
but if the percentage of Filipino ownership in the corporation or partnership is less than 60%, Heherson Alvarez of the Department of Environment and Natural Resources (DENR), seeking
42
cancellation of the CAMC FTAA for the primary reason that Republic Act 7942 and its Implementing Also affirmed by the Court was the legal standing of petitioners to raise the constitutionality of the
Rules and Regulations (IRR)-Department Administrative Order (DAO) No. 96-40 were questioned FTAA, as they had alleged personal and substantial injury from the mining activities of
unconstitutional. This letter was referred to the panel of arbitrators of the Mines and Geosciences CAMC. Likewise, they were under an imminent threat of being displaced from their landholdings as a
Bureau (MGB), Regional Office No. 2, Tuguegarao, Cagayan, for further action. The MGB eventually result of the implementation of the questioned FTAA.
rejected the demand.
Substantive Issues
Petitioners thus filed the present Petition for Prohibition and Mandamus. They prayed that the Court
issue an order (1) enjoining public respondent from acting on any application for an FTAA; (2) declaring Validity of Section 76 of
the Philippine Mining Act of 1995 and its IRR unconstitutional; and (3) canceling the FTAA issued to Republic Act 7942 and DAO 96-40
CAMC.
Petitioners claimed that Section 76 of Republic Act 7942 and DAO 96-40 were unconstitutional
The Issues for allowing the unlawful and unjust taking of private property for a private purpose. The Court,
however, rejected that claim. Underscoring the history of mining laws, it explained that their evolution
The parties raised two sets of issues. The first was procedural: Was the issue of eminent domain a gave a positive indication that mining operators who were qualified to own lands were granted the
justiciable controversy? Did petitioners have a legal standing to file the case? authority to exercise eminent domain for the entry, acquisition, and use of private lands in areas open to
mining operations.
The second set of issues was substantive in character: Did Section 76 of Republic Act 7942 allow
the unlawful taking of private property for private purpose? Did the assailed law and its IRR encroach Considering that Section 1 of PD 512 granted qualified mining operators that authority, which was
on the power of the trial courts to determine just compensation, inasmuch as that determination was deemed incorporated in Section 76 of Republic Act 7942, the inescapable conclusion was that the latter
cognizable only by the panel of arbitrators? Did the Mining Act make it possible for FTAA contracts to was a “taking” provision.
cede over to a fully foreign-owned corporation full control and management of mining enterprises,
resulting in the alleged reduction of the State to a passive regulator, dependent on submitted plans and By this conclusion, the Court did not mean, though, that Section 76 was unconstitutional for allowing the
reports and having only weak review and audit powers? Were foreign corporations barred from making taking of private property without a determination of public use and the payment of just
decisions on the conduct of operations and the management of the mining project? Were service compensation. Concededly, for a taking to be valid, it must be for public use. The mining industry was
contracts prohibited? deemed to play a pivotal role in the economic development of the country and to be a vital tool in the
government’s emphasis on accelerated recovery. It was therefore an industry that yielded public
The Court’s Decision, penned by Justice Minita V. Chico-Nazario,[1] brushed aside the procedural benefit.
objections and answered all the substantive questions in the negative.
The Court rejected petitioners’ claim that the State’s discretion to decide when to take private property
Procedural Issues had been contractually reduced by the CAMC FTAA. Moreover, it held that the assailed agreement had
laid down the ways and means by which a foreign contractor, disqualified from owning land, could
Respondents maintained that the issue of eminent domain was not ripe for adjudication, because identify for the government which specific surface areas within the FTAA contract area were to be
petitioners failed to allege that the latter’s properties had actually been taken by CAMC. Neither did acquired for the mine infrastructure. Through a voluntary transaction, the government would acquire
petitioners allege that their property rights had been endangered or were in danger on account of the ownership of those areas on behalf of the contractor, which would then be able to proceed with the full
corporation’s FTAA. Dismissing this issue, the Court held that there was a live controversy involving a implementation of the agreement.
clash of legal rights, because Republic Act 7942 had been enacted, DAO 96-40 approved, and FTAAs
entered into. Eminent domain was not yet called for at this stage, since there were still various avenues by which
surface rights could be acquired other than expropriation. The FTAA provision under attack merely
The FTAA holders were already operating in various provinces of the country. Among them was facilitated the implementation of the provision in question and thus shielded CAMC from a charge of
CAMC, which operated in various provinces of Nueva Vizcaya and Quirino. As a result of the CAMC violating the Anti-Dummy Law. Also, the Mining Law and its implementing rules provided for the
FTAA in those places, numerous individuals, including petitioners, were in danger of being ousted from payment of just compensation for private properties to be expropriated.
their landholdings. In this light, the Court could not await the adverse consequences of the law in order
to consider the controversy actual and ripe for judicial intervention. Actual eviction of the landowners The Power of Courts to
and occupants did not have to happen for this Court to intervene. Determine Just Compensation

43
It was contended that the assailed law and order encroached on the power of the trial courts to to leave no doubt in anyone’s mind about their true intent. Evidently, there was a conscious and
determine just compensation in eminent domain cases, inasmuch as that determination was cognizable deliberate decision on their part to avoid the use of restrictive wording that bespoke an intent not to use
only by a panel of arbitrators. the questioned expression in an exclusionary and limiting manner.

The Mining Act did not prevent the courts from taking cognizance of expropriation cases. The Service Contracts Not Prohibited
disagreement in Section 107 that petitioner referred to did not involve any exercise of eminent domain.
Rather, it contemplated a situation in which the permit holders were allowed entry into the lands of Petitioner contended that the service contract regime under the previous Constitution was
surface owners, and in which disagreement ensued as regards the proper compensation for the permitted expressly prohibited under the present one. “Service contracts,” the term found in the 1973 Constitution,
entry and use of the private lands. Noticeably, the provision pointed to a voluntary, not to an was later deleted to avoid the circumvention of constitutional prohibitions prevalent in the 1987 Charter.
involuntary, sale or transaction.
The Court brushed aside this contention. The term used earlier had not been carried over to the
The law conferred upon a panel of arbitrators the authority to decide cases in which permit holders were 1987 Constitution, which bore no categorical statement banning service contracts from mining
refused entry by owners, occupants, and concessionaires, thus necessitating an involuntary taking. The activities. This mere fact did not mean that service contracts, as understood in the 1973 Charter, were
grant of authority did not imply, though, that the determination of just compensation by the arbitrators or eradicated from the present Constitution. The latter still allowed the continued use of these contracts
the mines adjudication board was final and conclusive; the determination was only preliminary, unless with foreign corporations that would invest in and operate and manage extractive enterprises, subject to
accepted by all the parties concerned. There was nothing wrong with this procedure, because the the full control of the State. This time however, safety measures would be put in place to prevent the
original and exclusive jurisdiction of the courts to determine just compensation remained intact, despite abuses of the past regime. At bottom, the Court reiterated the doctrinal pronouncements in La Bugal.
the preliminary determination made by the administrative agency.

Sufficient State Control


Over Mining Operations

Petitioners asserted that the law, the implementing regulations, and the CAMC FTAA had ceded
beneficial ownership of mineral resources to the foreign contractor. Holding that this matter had already
been settled in La Bugal-B’laan Tribal Association v. Ramos,[2] the Court emphasized that the FTAA
contractor was not free to do whatever it pleased and get away with it; on the contrary, the contractor
would have to follow the government line if it wanted to stay in the enterprise. The law and its IRR had
vested in the government more than a sufficient degree of control and supervision over the conduct of
mining operations.

Proper Interpretation of
“Agreements Involving Either
Technical or Financial Assistance”

Petitioners maintained that the Constitution barred aliens and foreign-owned corporations from
entering into direct arrangements with the government. Such arrangements included agreements
involving co-production, joint venture, and production sharing. Moreover, the participation of foreign-
owned corporations in large-scale exploration was limited to agreements for either financial or technical
assistance only.

In discrediting this argument, the Court again cited La Bugal B’laan. The use of the word
“involving” in the phrase “agreements involving either technical or financial assistance” implied that
the agreements with foreign corporations were not limited to mere financial and technical assistance. If
the real intention of the drafters was to confine foreign corporations to financial or technical assistance
and nothing more, their language would have certainly been so unmistakably restrictive and stringent as

44

Potrebbero piacerti anche